/
OU Shoulder and Elbow OITE Review OU Shoulder and Elbow OITE Review

OU Shoulder and Elbow OITE Review - PowerPoint Presentation

olivia-moreira
olivia-moreira . @olivia-moreira
Follow
355 views
Uploaded On 2020-01-10

OU Shoulder and Elbow OITE Review - PPT Presentation

OU Shoulder and Elbow OITE Review September 23 2015 Betsy M Nolan MD 1 CONFIDENTIAL A 45yearold man who had gout in his foot 2 years ago has a 3day history of elbow pain without an injury The pain is diffuse constant and worse with any movement Examination shows motion from 20 degrees to 9 ID: 772398

pubmed shoulder pmid elbow shoulder pubmed elbow pmid confidential surg treatment arthroscopic repair bone cuff rotator preferred joint response

Share:

Link:

Embed:

Download Presentation from below link

Download Presentation The PPT/PDF document "OU Shoulder and Elbow OITE Review" is the property of its rightful owner. Permission is granted to download and print the materials on this web site for personal, non-commercial use only, and to display it on your personal computer provided you do not modify the materials and that you retain all copyright notices contained in the materials. By downloading content from our website, you accept the terms of this agreement.


Presentation Transcript

OU Shoulder and Elbow OITE Review September 23, 2015Betsy M. Nolan MD 1 CONFIDENTIAL

A 45-year-old man who had gout in his foot 2 years ago has a 3-day history of elbow pain without an injury. The pain is diffuse, constant, and worse with any movement. Examination shows motion from 20 degrees to 90 degrees. There is no erythema around his elbow, he has no fever, and a sensory and motor examination of his arm is unremarkable. Radiographs only show an effusion. The patient’s uric acid level is within defined limits. What is the next diagnostic step ? 1- Elbow joint aspiration 2- MRI scan 3- Splint for 2 weeks and repeat examination 4- Sedimentation rate and C-reactive protein level 2 CONFIDENTIAL

PREFERRED RESPONSE: 1- Elbow joint aspiration DISCUSSION The best way to make the diagnosis is to aspirate the fluid from the joint and send it to the laboratory for a cell count and crystal search. This will allow for the diagnosis of an infection, gout, or pseudogout. An MRI scan will confirm the examination finding of an effusion, but it will not reveal the cause of an inflammatory effusion. If the patient has chronic gout, an MRI scan or ultrasound can aid in diagnosis if gout tophi are seen. A splint will help relieve the pain from the effusion and the effusion may resolve on its own, but it will not contribute to a diagnosis. Sedimentation rate and C-reactive protein level will help to rule out an infection, but they are not as diagnostic as an aspiration. RECOMMENDED READINGS Orzechowski NM, Mason TG. Seronegative inflammatory arthritis. In: Morrey BF, Sanchez-Sotelo J, eds. The Elbow and Its Disorders. 4th ed. Philadelphia, PA: Saunders-Elsevier; 2009:1039-1041. de Ávila Fernandes E, Kubota ES, Sandim GB, Mitraud SA, Ferrari AJ, Fernandes AR. Ultrasound features of tophi in chronic tophaceous gout. Skeletal Radiol. 2011 Mar;40(3):309-15. Epub 2010 Jul 31. PubMed PMID: 20676636. 3 CONFIDENTIAL

A 65-year-old man who underwent an uncomplicated reverse total shoulder arthroplasty ( rTSA ) to treat rotator cuff arthropathy 2 years ago has a routine follow-up visit in your clinic. A radiograph at 2-year followup is shown in Figure 2. He denies shoulder pain and dysfunction and constitutional symptoms, and his clinical examination findings are benign. Based upon the present radiologic evaluation, what is the next most appropriate step? 1- Revision rTSA 2- Conversion to hemiarthroplasty 3- Continued observation4- Infection work-up with screening labs and joint aspiration 4 CONFIDENTIAL

PREFERRED RESPONSE: 3- Continued observation DISCUSSION Based upon the patient’s clinical examination and symptoms, continued observation is appropriate. The remaining options are not indicated. The radiograph reveals scapular notching, one of the more common complications specific to rTSA. Notching is caused by repeated contact between the humeral component and/or humerus and the inferior pillar of the scapular neck. Generation of particulate debris from this interaction can result in osteolysis with the potential for screw and base plate failure. The overall incidence of notching has been reported to be between 51% and 96%. This nearly ubiquitous finding has been attributed to implant positioning, altered glenoid and humeral anatomy, and duration of implantation. Recent studies that indicate increased lateral offset, increased glenosphere size, and inferior positioning of the base plate may reduce the incidence of scapular notching. RECOMMENDED READINGS Cheung E, Willis M, Walker M, Clark R, Frankle MA. Complications in reverse total shoulder arthroplasty. J Am Acad Orthop Surg. 2011 Jul;19(7):439-49. Review. PubMed PMID: 21724923.View Abstract at PubMed Lévigne C, Boileau P, Favard L, Garaud P, Molé D, Sirveaux F, Walch G. Scapular notching in reverse shoulder arthroplasty. J Shoulder Elbow Surg. 2008 Nov-Dec;17(6):925-35. Epub 2008 Jun 16. PubMed PMID: 18558499.View Abstract at PubMed Gutiérrez S, Levy JC, Frankle MA, Cuff D, Keller TS, Pupello DR, Lee WE 3rd. Evaluation of abduction range of motion and avoidance of inferior scapular impingement in a reverse shoulder model. J Shoulder Elbow Surg. 2008 Jul-Aug;17(4):608-15. Epub 2008 Mar 6. PubMed PMID: 18325795.5CONFIDENTIAL

A 61-year-old right-hand-dominant woman sustains a fall down 3 stairs, resulting in a left anteroinferior dislocation and noncomminuted greater tuberosity fracture. A closed glenohumeral reduction with intravenous sedation is performed in the emergency department. After reduction, the greater tuberosity fragment remains displaced by 2 mm. What is the most appropriate treatment? 1- Open reduction internal fixation with transosseous sutures 2- Arthroscopic fixation using a suture bridge technique 3- Nonsurgical treatment with early passive range of motion 4- Nonsurgical treatment with sling immobilization for 4 weeks 6 CONFIDENTIAL

PREFERRED RESPONSE: 3- Nonsurgical treatment with early passive range of motion DISCUSSION Greater tuberosity fractures and rotator cuff tears associated with a traumatic dislocation are more commonly seen in women older than age 60. Greater tuberosity fractures that are displaced less than 5 mm in the general population and less than 3 mm in laborers and professional athletes can be treated successfully without surgery. Early passive range of motion is important to avoid the complication of stiffness. RECOMMENDED READINGS George MS. Fractures of the greater tuberosity of the humerus. J Am Acad Orthop Surg. 2007 Oct;15(10):607-13. Review. PubMed PMID: 17916784.View Abstract at PubMed Platzer P, Kutscha-Lissberg F, Lehr S, Vecsei V, Gaebler C. The influence of displacement on shoulder function in patients with minimally displaced fractures of the greater tuberosity. Injury. 2005 Oct;36(10):1185-9. Epub 2005 Jun 16. PubMed PMID: 15963996.View Abstract at PubMed Mattyasovszky SG, Burkhart KJ, Ahlers C, Proschek D, Dietz SO, Becker I, Müller-Haberstock S, Müller LP, Rommens PM. Isolated fractures of the greater tuberosity of the proximal humerus: a long-term retrospective study of 30 patients . Acta Orthop. 2011 Dec;82(6):714-20. doi: 10.3109/17453674.2011.618912. Epub 2011 Sep 6. PubMed PMID: 21895502.7CONFIDENTIAL

CLINICAL SITUATION FOR QUESTIONS 5 AND 6 A 40-year-old riveter who works in a manufacturing plant experiences gradual onset of anterior shoulder pain that started 4 months ago, and he now has a constant ache in his shoulder. His pain wakes him at night and is worsened by lifting anything at shoulder height. He does not have any radiation of pain, and neck motion does not aggravate his pain. He stopped doing riveting work 1 month ago, but the pain did not improve. Examination shows normal motion and strength, a positive impingement sign, and tenderness over the anterior greater tuberosity. His sulcus sign is negative, and his Yergason and speed test findings are negative. He has normal scapular mechanics. Question 5 What is the next most appropriate step to confirm the diagnosis? 1- Radiograph 2- MRI scan 3- Subacromial injection with lidocaine 4- Ultrasound 8 CONFIDENTIAL

PREFERRED RESPONSE: 3- Subacromial injection with lidocaine 9CONFIDENTIAL

Question 6He started physical therapy while continuing light duty at work. Eight weeks later, his pain remained unchanged. An MRI scan is shown in Figure 5. What histologic changes are likely to be found in the supraspinatus tendon? 1- Disorganized collagen fibers with mucoid degeneration 2- Disorganized collagen fibers and acute inflammatory cells 3- Normal tendon fibers infiltrated with capillary proliferation 4- Normal tendon fibers infiltrated with acute inflammatory cells 10 CONFIDENTIAL

PREFERRED RESPONSE: 1- Disorganized collagen fibers with mucoid degeneration DISCUSSION This patient has impingement syndrome based on the history and examination. The best way to confirm the diagnosis is by performing a subacromial injection with lidocaine, which is also called a Neer impingement test. If the pain is relieved, the patient’s pain is coming from the subacromial space. An MRI scan would not confirm the diagnosis of impingement, although it can aid in diagnosis of other causes of anterior shoulder pain such as a rotator cuff tear. This patient has normal rotator cuff strength, so that diagnosis is less likely. A radiograph can show acromial morphology, which would support the diagnosis of impingement, but it does not rule out impingement if the radiograph findings are normal. Ultrasound would not support the diagnosis of impingement, but, like an MRI scan, it can reveal pathologies other than impingement. The MRI scan shows a supraspinatus tendon with changes consistent with tendinopathy, which is defined by disorganized collagen fibers with mucoid degeneration on the microscopic level. Although there are always exceptions, most tendinopathy occurs without inflammatory cells or capillary proliferation. RECOMMENDED READINGS Park HB, Yokota A, Gill HS, El Rassi G, McFarland EG. Diagnostic accuracy of clinical tests for the different degrees of subacromial impingement syndrome. J Bone Joint Surg Am. 2005 Jul;87(7):1446-55. PubMed PMID: 15995110.View Abstract at PubMed Khan KM, Cook JL, Bonar F, Harcourt P, Astrom M. Histopathology of common tendinopathies. Update and implications for clinical management. Sports Med. 1999 Jun;27(6):393-408. Review. PubMed PMID: 10418074. View Abstract at PubMed Lauder TD. Musculoskeletal disorders that frequently mimic radiculopathy. Phys Med Rehabil Clin N Am. 2002 Aug;13(3):469-85. Review. PubMed PMID: 12380546.View Abstract at PubMed Cannon DE, Dillingham TR, Miao H, Andary MT, Pezzin LE. Musculoskeletal disorders in referrals for suspected cervical radiculopathy. Arch Phys Med Rehabil. 2007 Oct;88(10):1256-9. PubMed PMID: 17908566.11CONFIDENTIAL

CLINICAL SITUATION FOR QUESTIONS 7 THROUGH 9 A 26-year-old man has the chief complaint of right shoulder instability. He underwent an uncomplicated arthroscopic Bankart repair following an injury sustained while playing high school football. His condition was stable for 7 years, but he redislocated his shoulder in a fall 6 months ago. He describes weekly anterior instability events that he can reduce on his own. Radiographs reveal a located glenohumeral joint, but a Hill-Sachs lesion is noted. A CT scan reveals a 20% anteroinferior glenoid deficiency and a Hill-Sachs lesion involving 20% of the humeral head. Question 7What is the best surgical option for this patient? 1- Coracoid transfer 2- Open Bankart repair3- Revision arthroscopic Bankart repair4- Arthroscopic remplissage procedure 12 CONFIDENTIAL

PREFERRED RESPONSE: 1- Coracoid transfer 13 CONFIDENTIAL

Question 8 What is the best indication to treat a Hill-Sachs lesion? 1- A lesion involving 20% of the humeral head that does not engage on examination2- A lesion involving 25% of the humeral head that remains located following instability repair 3- A lesion involving 30% of the humeral head that engages on examination4- A lesion involving 40% of the humeral head with recurrent glenohumeral instability 14 CONFIDENTIAL

PREFERRED RESPONSE: 4- A lesion involving 40% of the humeral head with recurrent glenohumeral instability 15 CONFIDENTIAL

Question 9 What is the most likely predictor of postsurgical pain following a coracoid transfer procedure for recurrent shoulder instability? 1- Suboptimal graft placement2- Pain before surgery 3- Progression of osteoarthritis4- Previous surgical treatment 16 CONFIDENTIAL

PREFERRED RESPONSE: 2- Pain before surgery DISCUSSION Recurrent glenohumeral instability with anteroinferior glenoid bone loss is best treated with a coracoid transfer. Schmid and associates reported on 49 patients with these lesions and recurrent instability who underwent coracoid transfer procedures. Good and excellent outcomes were reported in 88% of patients, and there were no instances of recurrent instability. With anteroinferior glenoid bone loss, capsular procedures—whether open or arthroscopic—are unlikely to provide stability. Remplissage is a procedure designed to address Hill-Sachs lesions rather than glenoid defects. The absolute indications to treat Hill-Sachs lesions surgically include a lesion associated with a humeral head fracture, a lesion involving more than 30% of the humeral head with chronic instability, and reverse Hill-Sachs lesions involving more than 20% of the articular surface with symptoms of posterior instability. Lesions involving 20% to 35% with or without engagement on examination are relative indications, as are lesions exceeding 10% that do not remain centered in the glenoid following arthroscopic stabilization. In Schmid and associates’ series of coracoid transfers for recurrent instability and anterior glenoid deficiency, patients who reported pain before surgery were 20 times more likely to have pain after surgery that compromised the functional outcome. Optimal graft placement correlated with better functional outcomes and less progression of arthrosis, but not with pain. Consequently, poor graft position, arthritis progression, and prior surgical treatment are not as consistently predictive of pain after surgery. RECOMMENDED READINGS Schmid SL, Farshad M, Catanzaro S, Gerber C. The Latarjet procedure for the treatment of recurrence of anterior instability of the shoulder after operative repair: a retrospective case series of forty-nine consecutive patients. J Bone Joint Surg Am. 2012 Jun 6;94(11):e75. doi: 10.2106/JBJS.K.00380. PubMed PMID: 22637215.View Abstract at PubMed Provencher MT, Frank RM, Leclere LE, Metzger PD, Ryu JJ, Bernhardson A, Romeo AA. The Hill-Sachs lesion: diagnosis, classification, and management. J Am Acad Orthop Surg. 2012 Apr;20(4):242-52. doi: 10.5435/JAAOS-20-04-242. Review. PubMed PMID: 22474094.View Abstract at PubMed Boileau P, O'Shea K, Vargas P, Pinedo M, Old J, Zumstein M. Anatomical and functional results after arthroscopic Hill-Sachs remplissage. J Bone Joint Surg Am. 2012 Apr 4;94(7):618-26. doi: 10.2106/JBJS.K.00101. PubMed PMID: 22488618.17CONFIDENTIAL

Question 10 A 45-year-old woman with diabetes has a 3-month history of left shoulder pain and motion loss unrelated to trauma. She previously underwent treatment with nonsteroidal anti-inflammatory medication and a home stretching program, experiencing minimal relief of her symptoms. Examination reveals loss of passive external rotation, abduction, and forward elevation without reduction in strength. Radiograph findings are normal. What is the most appropriate next step? 1- MRI scan2- Cortisone injection therapy with continued physical therapy (PT) 3- Closed manipulation under anesthesia 4- Arthroscopic release with manipulation under anesthesia 18 CONFIDENTIAL

PREFERRED RESPONSE: 2- Cortisone injection therapy with continued physical therapy (PT) DISCUSSION Based upon the duration of symptoms and clinical presentation, this patient would benefit from cortisone injection therapy and continued PT. Adhesive capsulitis is most commonly an idiopathic process that results in joint pain and loss of motion from capsular contracture. It affects approximately 2% to 5% of the general population. The process typically affects middle-age women. There are secondary causes such as previous trauma and fractures as well as associated medical conditions such as diabetes, stroke, and cardiac and thyroid disease. Debate remains as to whether there is a genetic predisposition for the development of adhesive capsulitis despite increased frequency noted in twin studies. Although the underlying etiology and pathophysiology are not well understood, the consensus is that synovial inflammation and capsular fibrosis result in pain and joint volume loss. It is hypothesized that in patients with diabetes, an increased rate of glycosylation and cross-linking of the shoulder capsule raises the incidence of frozen shoulder. For this patient, history reveals a short course of symptoms that did not improve with nonsurgical modalities. Clinically, the patient has reduced passive range of motion, particularly with external and internal rotation and forward elevation. Radiographs are usually obtained to exclude other causes of shoulder pain such as glenohumeral arthrosis, malignancy, calcific tendonitis, impingement, and acromioclavicular degeneration. If pain and stiffness persist beyond 6 months, closed manipulation may be an option. Complications associated with this modality may include humerus fracture, dislocation, hematoma, rotator cuff and labral tears, and brachial plexus injury. Some surgeons advocate arthroscopic capsular release to allow for examination of concomitant pathology and controlled release of capsular tissue, with the potential for reduced required force when performing the manipulation portion of the procedure. This modality may be appropriate after an initial treatment with PT. Controversy remains as to whether posterior capsular release should be performed routinely because studies have shown outcomes to be similar with anterior and combined approaches. Therapy should be initiated early after intervention, with some surgeons advocating admission to the hospital with inpatient therapy for pain management and compliance. RECOMMENDED READINGS Neviaser AS, Neviaser RJ. Adhesive capsulitis of the shoulder. J Am Acad Orthop Surg. 2011 Sep;19(9):536-42. Review. PubMed PMID: 21885699.View Abstract at PubMed Hsu JE, Anakwenze OA, Warrender WJ, Abboud JA. Current review of adhesive capsulitis. J Shoulder Elbow Surg. 2011 Apr;20(3):502-14. Epub 2010 Dec 16. Review. PubMed PMID: 21167743.View Abstract at PubMed Le Lievre HM, Murrell GA. Long-term outcomes after arthroscopic capsular release for idiopathic adhesive capsulitis. J Bone Joint Surg Am. 2012 Jul 3;94(13):1208-16. PubMed PMID: 22760389.View Abstract at PubMed Yian EH, Contreras R, Sodl JF. Effects of glycemic control on prevalence of diabetic frozen shoulder. J Bone Joint Surg Am. 2012 May 16;94(10):919-23. PubMed PMID: 22617920.19CONFIDENTIAL

A 42-year-old woman has a posterior elbow dislocation. Closed reduction is performed, and the elbow appears stable under fluoroscopic examination. Further treatment should consist of 1- early mobilization only. 2- surgical repair or reconstruction of the lateral collateral ligament (LCL) and the medial collateral ligament (MCL). 3- active range of motion in a hinged brace with a range of 30 degrees to 120 degrees.4- application of a hinged external fixator with early mobilization. 20 CONFIDENTIAL

PREFERRED RESPONSE: 1- early mobilization only. DISCUSSION This is a simple (no associated fracture) elbow dislocation. Such dislocations can be treated with closed reduction followed by mobilization after 5 to 7 days to avoid stiffness, provided the elbow is stable through a full arc of motion at the time of reduction. If the elbow is unstable but has a short arc of stability, then using a hinged brace in the stable arc may be considered. (Note: It may be necessary to splint the elbow in pronation if the MCL is intact and the LCL is disrupted or in supination if the LCL is intact but the MCL disrupted.) Surgical reconstruction of the LCL and MCL may be required only if the elbow does not have a stable arc at the time of reduction. If unstable after reconstruction, application of a hinged external fixator may be considered. RECOMMENDED READINGS Glaser D, Armstrong A. Elbow and forearm trauma. In: Flynn J ed. Orthopaedic Knowledge Update 10. Rosemont, IL: American Academy of Orthopaedic Surgeons; 2011:325-341. Anakwe RE, Middleton SD, Jenkins PJ, McQueen MM, Court-Brown CM. Patient-reported outcomes after simple dislocation of the elbow. J Bone Joint Surg Am. 2011 Jul 6;93(13):1220-6. doi: 10.2106/JBJS.J.00860. PubMed PMID: 21776575.View Abstract at PubMed Mehlhoff TL, Noble PC, Bennett JB, Tullos HS. Simple dislocation of the elbow in the adult. Results after closed treatment. J Bone Joint Surg Am. 1988 Feb;70(2):244-9. PubMed PMID: 3343270. 21 CONFIDENTIAL

A 65-year-old woman has 4 months of atraumatic shoulder pain persisting despite physical therapy and activity modification. She has normal range of motion, and an MRI scan reveals a 10% thickness partial articular supraspinatus tear. 1- Physical therapy and activity modification 2- Corticosteroid injection 3- Arthroscopic glenohumeral capsular release 4- Arthroscopic superior labrum anterior to posterior (SLAP) repair 5- Arthroscopic subacromial decompression and rotator cuff debridement 6- Arthroscopic subacromial decompression and rotator cuff repair 22 CONFIDENTIAL

PREFERRED RESPONSE: 2- Corticosteroid injection 23 CONFIDENTIAL

A 35-year-old mechanic has 6 months of shoulder pain following an axial traction work-related injury. His pain has persisted despite extensive physical therapy and work restrictions. A noncontrast MRI scan shows a 90% partial bursal-sided supraspinatus tear. 1- Physical therapy and activity modification 2- Corticosteroid injection 3- Arthroscopic glenohumeral capsular release 4- Arthroscopic superior labrum anterior to posterior (SLAP) repair 5- Arthroscopic subacromial decompression and rotator cuff debridement 6- Arthroscopic subacromial decompression and rotator cuff repair 24 CONFIDENTIAL

PREFERRED RESPONSE: 6- Arthroscopic subacromial decompression and rotator cuff repair 25CONFIDENTIAL

A 25-year-old 100% service connected Veteran has a type II SLAP lesion, with 4 weeks of new-onset atraumatic shoulder pain. 1- Physical therapy and activity modification 2- Corticosteroid injection 3- Arthroscopic superior labrum anterior to posterior (SLAP) repair 4- Arthroscopic subacromial decompression and rotator cuff debridement 5- Email your shipmates to ask for their help. 26 CONFIDENTIAL

27 CONFIDENTIAL

PREFERRED RESPONSE: 1- Physical therapy and activity modification 28 CONFIDENTIAL

A 75-year-old man has had 8 months of persistent, atraumatic shoulder pain. He had transient improvement with physical therapy and a subacromial corticosteroid injection. MRI scan shows a 25% partial articular supraspinatus/subscapularis tear and significant subacromial bursal inflammation. 1- Physical therapy and activity modification 2- Corticosteroid injection 3- Arthroscopic glenohumeral capsular release 4- Arthroscopic superior labrum anterior to posterior (SLAP) repair 5- Arthroscopic subacromial decompression and rotator cuff debridement 6- Arthroscopic subacromial decompression and rotator cuff repair 29CONFIDENTIAL

PREFERRED RESPONSE: 5- Arthroscopic subacromial decompression and rotator cuff debridement DISCUSSION The management of partial rotator cuff tears depends upon many factors, including patient age, symptom onset and severity, prior treatment, physical limitation(s) based on history and examination, and the extent of structural involvement based upon detailed imaging (typically MRI or ultrasound). The incidence of a partial rotator cuff tear, based on imaging with MRI or ultrasound, is high in patients ages 60 years and older. Patients beyond age 60 with either mild or new-onset symptoms with preserved active and passive range of motion are excellent candidates for physical therapeutic intervention and avoidance of exacerbating activities, particularly when MRI scan or ultrasound reveal less than 50% tendon involvement. Partial rotator cuff tears are also common in the dominant arm of overhead athletes, and frequently respond to nonsurgical treatment, as well. These types of partial rotator cuff tears often are seen in combination with superior labral pathology. Rotator cuff repair usually is recommended for patients with tears that involve more than 50% of tendon thickness who have failed a reasonable attempt at nonsurgical management, particularly patients who are young and have high activity demands. Partial-sided bursal tears may be more symptomatic and respond well to surgical repair, but patients may not do as well with subacromial decompression alone. Global loss of glenohumeral motions is consistent with adhesive capsulitis. Such patients are initially treated with therapy that emphasizes range of motion, usually incorporating a home exercise program. Finally, subacromial decompression may be considered for patients with low-grade partial articular rotator cuff tears that have failed nonsurgical management and substantially interfere with daily and/or recreational activities. 30 CONFIDENTIAL

RECOMMENDED READINGS Sher JS, Uribe JW, Posada A, Murphy BJ, Zlatkin MB. Abnormal findings on magnetic resonance images of asymptomatic shoulders. J Bone Joint Surg Am. 1995 Jan;77(1):10-5. PubMed PMID: 7822341.View Abstract at PubMed Weber SC. Arthroscopic debridement and acromioplasty versus mini-open repair in the treatment of significant partial-thickness rotator cuff tears. Arthroscopy. 1999 Mar;15(2):126-31. PubMed PMID: 10210067. View Abstract at PubMed Cordasco FA, Backer M, Craig EV, Klein D, Warren RF. The partial-thickness rotator cuff tear: is acromioplasty without repair sufficient? Am J Sports Med. 2002 Mar-Apr;30(2):257-60. PubMed PMID: 11912097.View Abstract at PubMed Connor PM, Banks DM, Tyson AB, Coumas JS, D'Alessandro DF. Magnetic resonance imaging of the asymptomatic shoulder of overhead athletes: a 5-year follow-up study. Am J Sports Med. 2003 Sep-Oct;31(5):724-7. PubMed PMID: 12975193.View Abstract at PubMed Yamaguchi K, Ditsios K, Middleton WD, Hildebolt CF, Galatz LM, Teefey SA. The demographic and morphological features of rotator cuff disease. A comparison of asymptomatic and symptomatic shoulders. J Bone Joint Surg Am. 2006 Aug;88(8):1699-704. PubMed PMID: 16882890.View Abstract at PubMed Loeffler BJ, Brown SL, D'Alessandro DF, Fleischli JE, Connor PM. Incidence of False Positive Rotator Cuff Pathology in MRIs of Patients with Adhesive Capsulitis. Orthopedics. 2011 May 18;34(5):362. doi: 10.3928/01477447-20110317-14. PubMed PMID: 21598899.View Abstract at PubMed Pedowitz RA, Yamaguchi K, Ahmad CS, Burks RT, Flatow EL, Green A, Iannotti JP, Miller BS, Tashjian RZ, Watters WC 3rd, Weber K, Turkelson CM, Wies JL, Anderson S, St Andre J, Boyer K, Raymond L, Sluka P, McGowan R; American Academy of Orthopaedic Surgeons. Optimizing the management of rotator cuff problems. J Am Acad Orthop Surg. 2011 Jun;19(6):368-79. PubMed PMID: 21628648.31CONFIDENTIAL

The problem seen above is most likely associated with injury to what structure? 1- Inferior glenohumeral ligament 2- Acromioclavicular (AC) ligaments 3- Coracoclavicular ligaments 4- Coracoacromial ligament 5. Pocket protector 32 CONFIDENTIAL

PREFERRED RESPONSE: 3- Coracoclavicular ligaments DISCUSSION The radiograph shows an extra-articular distal clavicle fracture lateral to the clavicular attachment point of the coracoclavicular ligaments (conoid and trapezoid). However, unlike a scenario featuring a typical Neer type I fracture, the interval between coracoid and clavicle is clearly widened and there is marked fracture displacement. It is clear that the coracoclavicular ligaments must also be torn. The inferior glenohumeral ligament is important to glenohumeral joint stability, but has no effect on the relationship between clavicle and scapula. The AC ligaments are thickenings of the AC joint capsule. They have been shown to be responsible for 90% of anteroposterior stability of the AC joint. The coracoclavicular ligaments are responsible for 77% of stability for superior translation (as in this case). The coracoacromial ligament connects 2 parts of the scapula (coracoids and acromion) and is part of the arch that supports the rotator cuff. RECOMMENDED READINGS NEER CS 2nd. Fracture of the distal clavicle with detachment of the coracoclavicular ligaments in adults. J Trauma. 1963 Mar;3:99-110. PubMed PMID: 13937900.View Abstract at PubMed URIST MR. COMPLETE DISLOCATION OF THE ACROMIOCLAVICULAR JOINT. J Bone Joint Surg Am. 1963 Dec;45:1750-3. PubMed PMID: 14083156. View Abstract at PubMed Acus RW 3rd, Bell RH, Fisher DL. Proximal clavicle excision: an analysis of results. J Shoulder Elbow Surg. 1995 May-Jun;4(3):182-7. PubMed PMID: 7552675.33CONFIDENTIAL

34 CONFIDENTIAL

A 36-year-old right-hand-dominant man fell from his motorcycle and sustained the acute right upper extremity injury seen. At surgery, an open reduction and internal fixation of the ulna is performed along with attempted open reduction of the radiocapitellar joint. However, the radial head is slightly subluxed in flexion and redislocates with elbow extension below 90 degrees. What is the most appropriate treatment at this time? 1- Radial head resection 2- Casting in 90 degrees of flexion for 3 weeks, followed by reassessment of elbow stability3- Repair of the annular ligament 4- Revision open reduction and internal fixation of the ulnar fracture 35 CONFIDENTIAL

PREFERRED RESPONSE: 4- Revision open reduction and internal fixation of the ulnar fracture DISCUSSION This case is a variant of a type I Monteggia fracture according to the Bado classification with a segmental ulna fracture. In some cases, the radial head subluxation can be subtle, and missing this would lead to a poor outcome. In this case, the anterior radial head dislocation is obvious, but the segmental nature of the ulna fracture makes anatomic reduction difficult. The radial head usually spontaneously reduces once the ulna is anatomically reduced, and no surgical treatment to the lateral side is required. When this is not the case, a lateral approach and incision of the annular ligament may be required for reduction. If an open reduction of the radial head is unsuccessful, the problem is almost always residual malalignment of the ulna. Therefore, casting and annular ligament repair will not improve reduction. A radial head resection would eliminate the nonconcentric contact between radial head and capitellum , but would not be an appropriate treatment for this young patient who has an acute, correctable fracture deformity. RECOMMENDED READINGS Bado JL. The Monteggia lesion. Clin Orthop Relat Res. 1967 Jan-Feb;50:71-86. PubMed PMID: 6029027.View Abstract at PubMed Ring D, Jupiter JB, Waters PM. Monteggia fractures in children and adults. J Am Acad Orthop Surg. 1998 Jul-Aug;6(4):215-24. Review. PubMed PMID: 9682084. 36 CONFIDENTIAL

37 CONFIDENTIAL

Figure 19 is the radiograph of a 45-year-old right-hand-dominant man who has had a 2-day history of right shoulder pain, weakness, and a deformity involving the clavicle region after a fall from a scaffold during work activities. He was previously evaluated by his primary care physician and another orthopaedist. He has sought a second opinion regarding his treatment options. What is the most appropriate treatment for his injury? 1- Sling immobilization with continued observation2- Open reduction and plate fixation3- Open reduction, plate fixation, and application of an external bone stimulator 4- External fixation 38 CONFIDENTIAL

PREFERRED RESPONSE: 2- Open reduction and plate fixation DISCUSSION Midshaft clavicle fractures are relatively common and account for 3% to 10% of all adult fractures. Historical studies regarding nonsurgical treatment of displaced clavicle fractures indicated a low nonunion rate approaching 1%. Contemporary studies, however, suggest that the nonunion rate is much higher, reaching 15% to 20% with corresponding loss of shoulder strength and residual deficits at 6 months from date of injury. A recent meta-analysis by McKee and associates comparing nonsurgical treatment with a sling and surgical treatment with plate fixation concluded that initial fixation of displaced midshaft clavicle fractures demonstrated a positive effect on pain reduction at 3 weeks and improved Disabilities of the Arm, Shoulder, and Hand (DASH) scores at 6 weeks and 3 months after surgery. The choice of fixation remains a matter of debate because both plate and pin fixation have been used to achieve clavicle union. Intramedullary fixation of clavicle fractures historically has demonstrated an increased risk for pin tract infections and hardware removal attributable to local irritation from the implant. External bone stimulator use has not proven beneficial in effecting a reduction of nonunion rates. The most common complications noted with surgical intervention included local hardware irritation and wound infection. RECOMMENDED READINGS McKee RC, Whelan DB, Schemitsch EH, McKee MD. Operative versus nonoperative care of displaced midshaft clavicular fractures: a meta-analysis of randomized clinical trials. J Bone Joint Surg Am. 2012 Apr 18;94(8):675-84. Review. PubMed PMID: 22419410.View Abstract at PubMed Canadian Orthopaedic Trauma Society. Nonoperative treatment compared with plate fixation of displaced midshaft clavicular fractures. A multicenter, randomized clinical trial. J Bone Joint Surg Am. 2007 Jan;89(1):1-10. PubMed PMID: 17200303.View Abstract at PubMed Judd DB, Pallis MP, Smith E, Bottoni CR. Acute operative stabilization versus nonoperative management of clavicle fractures. Am J Orthop (Belle Mead NJ). 2009 Jul;38(7):341-5. PubMed PMID: 19714275.39CONFIDENTIAL

A 55-year-old woman develops posttraumatic arthritis in the elbow following a distal humerus fracture. What is the most likely mid-term (5-10 years after surgery) complication following semiconstrained total elbow arthroplasty (TEA)? 1- Bushing wear2- Infection3- Aseptic component loosening 4- Component fracture 40 CONFIDENTIAL

PREFERRED RESPONSE: 1- Bushing wear DISCUSSION TEA has been described for posttraumatic arthritis of the elbow and typically involves a young patient population with multiple previous operations on the affected elbow. Morrey and Schneeberger found aseptic component loosening to be uncommon (fewer than 10% of patients), and usually occurs more than 10 years after surgery. Prosthetic fracture, usually of the ulnar component, is also a late-term finding. Infection is the most common mode of early failure, but usually occurs within the first 5 years and has an overall rate of approximately 5%. Bushing wear has been reported as the most common cause of mechanical TEA failure in this population at intermediate-term followup . RECOMMENDED READINGS Schneeberger AG, Adams R, Morrey BF. Semiconstrained total elbow replacement for the treatment of post-traumatic osteoarthrosis . J Bone Joint Surg Am. 1997 Aug;79(8):1211-22. PubMed PMID: 9278082.View Abstract at PubMed Morrey BF, Schneeberger AG. Total elbow arthroplasty for posttraumatic arthrosis. Instr Course Lect. 2009;58:495-504. PubMed PMID: 19385558.View Abstract at PubMed Throckmorton T, Zarkadas P, Sanchez-Sotelo J, Morrey B. Failure patterns after linked semiconstrained total elbow arthroplasty for posttraumatic arthritis. J Bone Joint Surg Am. 2010 Jun;92(6):1432-41. doi: 10.2106/JBJS.I.00145. PubMed PMID: 20516319.41CONFIDENTIAL

CLINICAL SITUATION FOR QUESTIONS 21 AND 22 A 21-year-old rugby player has had the sensation of shoulder instability while making tackles for 3 years. Two years ago, he had an arthroscopic Bankart repair and capsulorrhaphy that used 3 suture anchors after dislocating his shoulder while making a tackle. This procedure required an emergency department sedated reduction. After this dislocation, he had paresthesias in his arm and a sense of weakness. His numbness eventually resolved. He did well after surgery until 2 weeks ago, when he again felt his shoulder dislocate while tackling and had an emergency department reduction. Question 21 What caused his recurrent instability? 1- The use of suture anchors in his repair 2- The physical therapy program after surgery 3- His age at the time of first surgery 4- His activity levels after surgery 5-Wearing packs when he was in the Army42CONFIDENTIAL

PREFERRED RESPONSE: 4- His activity levels after surgery 43 CONFIDENTIAL

Question 22 Numbness after his first dislocation was related to 1- intrasurgical traction on the musculocutaneous nerve. 2- residual interscalene blockade.3- ulnar neuropathy after sling use. 4- sensory axillary nerve palsy from his dislocation. 44 CONFIDENTIAL

PREFERRED RESPONSE: 4- sensory axillary nerve palsy from his dislocation. DISCUSSION Several studies have demonstrated the equivalency of arthroscopic Bankart repair to open surgery, but most studies have also demonstrated increased recurrence rates in patients who participate in collision sports such as rugby. Revision surgery with arthroscopy is unlikely to be durable, and in patients who have large glenoid defects, a coracoid process transfer (Latarjet or Bristow) is the preferred surgical treatment. Arthroplasty would not be indicated in a young and active patient. RECOMMENDED READINGS Neyton L, Young A, Dawidziak B, Visona E, Hager JP, Fournier Y, Walch G. Surgical treatment of anterior instability in rugby union players: clinical and radiographic results of the Latarjet-Patte procedure with minimum 5-year follow-up. J Shoulder Elbow Surg. 2012 Dec;21(12):1721-7. doi: 10.1016/j.jse.2012.01.023. Epub 2012 May 5. PMID 22565042.View Abstract at PubMed Robinson CM, Shur N, Sharpe T, Ray A, Murray IR. Injuries associated with traumatic anterior glenohumeral dislocations. J Bone Joint Surg Am. 2012 Jan 4;94(1):18-26. PMID: 22218378. 45 CONFIDENTIAL

Figure 23a Figure 23b Figure 23c Figure 23d Question 23 of 105 Figures 23a through 23d are the radiographs and MRI scans of a 30-year-old otherwise healthy man who sustained an anterior right shoulder dislocation while playing baseball. He requires a closed reduction under sedation at a local emergency department. He is placed into a shoulder immobilizer and referred to your office for further treatment. Upon inquiry, the patient states that he previously dislocated the shoulder twice within the last year while playing basketball. He demonstrates positive apprehension and speed tests. What is the most appropriate next treatment step? 1- Brief period of immobilization with initiation of therapy 2- Arthroscopic labral repair 3- Open capsular shift 4- Coracoid transfer 46 CONFIDENTIAL

PREFERRED RESPONSE: 2- Arthroscopic labral repair DISCUSSION By history, this patient has recurrent anterior instability. His radiographs do not indicate significant deficiency of the anterior glenoid, which would potentially require augmentation with a coracoid transfer to restore stability to the glenohumeral joint. An MRI scan reveals a displaced anteroinferior labral tear ( Bankart lesion) with extension into the biceps labral complex. An open capsular shift can address capsular redundancy, but an arthroscopic procedure will allow for examination of concomitant pathology and the ability to address the capsulolabral tear with reduced morbidity. To minimize this patient's redislocation risk with sports activities, an arthroscopic capsulolabral repair involving the anteroinferior and superior labrum is the most appropriate solution. Hantes and associates demonstrated that clinical outcomes are similar at 2-year followup in patients with combined anteroinferior and superior labral tears vs patients with isolated Bankart lesions when treated by arthroscopic means. RECOMMENDED READINGS Carreira DS, Mazzocca AD, Oryhon J, Brown FM, Hayden JK, Romeo AA. A prospective outcome evaluation of arthroscopic Bankart repairs: minimum 2-year follow-up. Am J Sports Med. 2006 May;34(5):771-7. PubMed PMID: 16627629 . Taylor DC, Arciero RA. Pathologic changes associated with shoulder dislocations. Arthroscopic and physical examination findings in first-time, traumatic anterior dislocations. Am J Sports Med. 1997 May-Jun;25(3):306-11. PubMed PMID: 9167808.Hantes ME, Venouziou AI, Liantsis AK, Dailiana ZH, Malizos KN. Arthroscopic repair for chronic anterior shoulder instability: a comparative study between patients with Bankart lesions and patients with combined Bankart and superior labral anterior posterior lesions. Am J Sports Med. 2009 Jun;37(6):1093-8. Epub 2009 Mar 13. PubMed PMID: 19286910.47CONFIDENTIAL

Figure 27 Figure 27 is the MRI scan of a 63-year-old man who has experienced 3 years of shoulder pain. He has had 2 fluoroscopically guided corticosteroid injections that provided him with temporary but significant relief. His primary care physician ordered an MRI scan because of his ongoing shoulder issues. His examination shows significant loss of motion in all planes but good motor strength. The best treatment at this point would include 1- hyaluronic acid injection. 2- physical therapy. 3- reverse total shoulder arthroplasty. 4- anatomic total shoulder arthroplasty. 48 CONFIDENTIAL

PREFERRED RESPONSE: 4- anatomic total shoulder arthroplasty. DISCUSSION Glenohumeral arthritis in this age group is best treated with total shoulder arthroplasty. It provides more durable and cost-effective relief than hemiarthroplasty. Hyaluronic acid has been demonstrated in some studies to be effective at improving pain in the short term, but is not approved by the U.S. Food and Drug Administration for use in the shoulder. Reverse total shoulder replacement is indicated in patients who have an irreparable rotator cuff tear. This image clearly shows the supraspinatus is intact. Physical therapy has not proven beneficial in the setting of end-stage glenohumeral arthritis. RECOMMENDED READINGS Izquierdo R, Voloshin I, Edwards S, Freehill MQ, Stanwood W, Wiater JM, Watters WC 3rd, Goldberg MJ, Keith M, Turkelson CM, Wies JL, Anderson S, Boyer K, Raymond L, Sluka P; Treatment of glenohumeral osteoarthritis. American Academy of Orthopedic Surgeons. J Am Acad Orthop Surg. 2010 Jun;18(6):375-82. PMID: 20511443. Mather RC 3rd, Watters TS, Orlando LA, Bolognesi MP, Moorman CT 3rd. Cost effectiveness analysis of hemiarthroplasty and total shoulder arthroplasty. J Shoulder Elbow Surg. 2010 Apr;19(3):325-34. PMID: 20303459 . Edwards TB, Kadakia NR, Boulahia A, Kempf JF, Boileau P, Némoz C, Walch G. A comparison of hemiarthroplasty and total shoulder arthroplasty in the treatment of primary glenohumeral osteoarthritis: results of a multicenter study. J Shoulder Elbow Surg. 2003 May-Jun;12(3):207-13. PMID: 12851570.Gartsman GM, Roddey TS, Hammerman SM. Shoulder arthroplasty with or without resurfacing of the glenoid in patients who have osteoarthritis. J Bone Joint Surg Am. 2000 Jan;82(1):26-34. PMID: 10653081.49CONFIDENTIAL

Figure 28 CLINICAL SITUATION FOR QUESTIONS 28 AND 29 Figure 28 is the radiograph of a 5 9-year-old woman who fell from a ladder 6 weeks ago when a tree her brother was cutting down accidentally hit the ladder . After being evaluated for additional trauma, t he emergency room physician ordered a VA “standard” shoulder series, which consists of an AP in neutral rotation, AP in external rotation and AP in internal rotation. The radiologist read this series as normal, and the patient was discharged home. She is now referred to your office for left shoulder pain and loss of external rotation. Question 28 What is the most likely diagnosis? 1- Rotator cuff tear 2- Adhesive capsulitis 3- Brachial plexus injury 4- Posterior shoulder dislocation 50 CONFIDENTIAL

PREFERRED RESPONSE: 4- Posterior shoulder dislocation 51 CONFIDENTIAL

Question 29 What is the best next step? 1- Physical therapy 2- CT scan 3- Arthroscopic capsular release 4- Arthroscopic Bankart repair 52 CONFIDENTIAL

PREFERRED RESPONSE: 2- CT scan DISCUSSION The patient has a posterior glenohumeral dislocation, as evidenced by the overlap on the initial radiograph. While posterior dislocations are rare, they can be overlooked. A CT scan will accurately show the lesion prior to a trip to the operating room. If a simple closed reduction is performed acutely and the arm is stable after the reduction, no further intervention may be needed and treatment can be successful with a 2-week period of immobilization for defects involving less than 30% of the humeral head. However, in this scenario, open reduction is likely and stabilization may require a modified McLaughlin procedure or other intervention to fill in the humeral defect. Younger male patients, those with a large humeral head defect, and those with seizure disorder may be at highest risk for recurrence. For treatment of chronic posterior dislocations, it may be necessary to perform shoulder arthroplasty to restore stability. Stiffness is attributable to articular incongruity; therefore, physical therapy and capsular release are inappropriate. RECOMMENDED READINGS Duralde XA, Fogle EF. The success of closed reduction in acute locked posterior fracture-dislocations of the shoulder. J Shoulder Elbow Surg. 2006 Nov-Dec;15(6):701-6. Epub 2006 Oct 19. PubMed PMID: 17055305.Robinson CM, Aderinto J. Posterior shoulder dislocations and fracture-dislocations. J Bone Joint Surg Am. 2005 Mar;87(3):639-50. Review. PubMed PMID: 15741636.Robinson CM, Seah M, Akhtar MA. The epidemiology, risk of recurrence, and functional outcome after an acute traumatic posterior dislocation of the shoulder. J Bone Joint Surg Am. 2011 Sep 7;93(17):1605-13. doi : 10.2106/JBJS.J.00973. PubMed PMID: 21915575. Sperling JW, Pring M, Antuna SA, Cofield RH. Shoulder arthroplasty for locked posterior dislocation of the shoulder. J Shoulder Elbow Surg. 2004 Sep-Oct;13(5):522-7. PubMed PMID: 15383808.53CONFIDENTIAL

A 40-year-old right-hand-dominant construction worker has a 3-month history of right shoulder weakness secondary to a fall from a ladder at work. He underwent nonsurgical treatment with anti-inflammatory medication, cortisone injections, and therapy, with minimal relief of his symptoms. A subsequent MRI scan indicates a 1-cm full-thickness supraspinatus tendon tear. He has been referred to your clinic for discussion of surgical intervention. The patient's nurse case manager is concerned that he may not be able to return to his preinjury level of activity at work, even with surgical intervention. You tell the nurse case manager that, on average, the patient will 1- be at increased risk for infection compared to patients without a Worker’s Compensation claim. 2- have significant functional improvement after rotator cuff repair that is less robust than that of patients without a Worker’s Compensation claim. 3- have pain relief that is equivalent to that of patients without a Worker’s Compensation claim. 4- return to work without restrictions within a 3-month time frame. 54 CONFIDENTIAL

PREFERRED RESPONSE: 2- have significant functional improvement after rotator cuff repair that is less robust than that of patients without a Worker’s Compensation claim. DISCUSSION Many factors have been associated with less-than-favorable outcomes following rotator cuff repair such as tear size, age at time of intervention, gender, fatty degeneration of rotator cuff musculature, and Worker’s Compensation status. Henn and associates performed a prospective study regarding rotator cuff repairs in a cohort of 125 patients to assess the factors that may affect outcome as measured with the Simple Shoulder Test (SST), Disabilities of the Arm, Shoulder, and Hand (DASH), Short Form-36 (SF-36), and Visual Analog Pain Scale (VAS). When confounding factors were controlled, Worker’s Compensation status was an independent predictor of poorer DASH scores. With the use of historical controls, Bhatia and associates concluded that the vast majority (89%) of workers who underwent an arthroscopic rotator cuff repair returned to their preoperative level of work at a mean time of 7.6 months. There was a trend toward decreased return to full duty with increased work demands before surgery (light, medium, and heavy duty), but this result did reach statistical significance. Alcohol consumption (more than 6 drinks per week) was the only factor to demonstrate an association with postoperative restricted work duty and increased rotator cuff repair failure. There is no evidence to support increased infection rates for rotator cuff repair in Worker’s Compensation patients. RECOMMENDED READINGS Bhatia S, Piasecki DP, Nho SJ, Romeo AA, Cole BJ, Nicholson GP, Boniquit N, Verma NN. Early return to work in workers' compensation patients after arthroscopic full-thickness rotator cuff repair. Arthroscopy. 2010 Aug;26(8):1027-34. Epub 2010 Jun 3. PubMed PMID: 20678699. Henn RF 3rd, Kang L, Tashjian RZ, Green A. Patients with workers' compensation claims have worse outcomes after rotator cuff repair. J Bone Joint Surg Am. 2008 Oct;90(10):2105-13. PubMed PMID: 18829907. 55CONFIDENTIAL

A 75-year-old man sustains an anterior dislocation of his reverse total shoulder arthroplasty. What activity places the arm in the position most commonly associated with reverse total shoulder dislocation? 1- Scratching the opposite shoulder 2- Pushing off an ipsilateral chair armrest to assist in standing up 3- Tying shoelaces on the contralateral foot 4- Brushing hair 56 CONFIDENTIAL

PREFERRED RESPONSE: 2- Pushing off an ipsilateral chair armrest to assist in standing up DISCUSSION Proper soft-tissue tension is critical to prevent instability of a reverse total shoulder implanted with the deltopectoral approach; dislocation of the prosthesis is exceedingly rare if the superior approach is employed. The arm position implicated in reverse total shoulder instability is extension, adduction, and internal rotation, such as pushing out of a chair. The other positions described do not involve extension of the shoulder. RECOMMENDED READINGS Gerber C, Pennington SD, Nyffeler RW. Reverse total shoulder arthroplasty. J Am Acad Orthop Surg. 2009 May;17(5):284-95. Review. PubMed PMID: Cheung E, Willis M, Walker M, Clark R, Frankle MA. Complications in reverse total shoulder arthroplasty. J Am Acad Orthop Surg. 2011 Jul;19(7):439-49. Review. PubMed PMID: 21724923.19411640.View Abstract at PubMed Cheung E, Willis M, Walker M, Clark R, Frankle MA. Complications in reverse total shoulder arthroplasty. J Am Acad Orthop Surg. 2011 Jul;19(7):439-49. Review. PubMed PMID: 21724923. View Abstract at PubMed Walch G, Wall B, Mottier F: Complication and revision of the reverse prosthesis: A multicenter study of 457 cases. In: Walch G, Boileau P, Mole P, Favard L, Levigne C, Sirveaux f, eds. Reverse Shoulder Arthroplasty: Clinical Results, Complications, Revision. Montpellier, France: Sauramps Médical; 2006:335-352.57CONFIDENTIAL

When performing an ulnar nerve decompression at the elbow, the surgeon must be aware of the 1- median nerve as it crosses the surgical field 6 cm proximal to the medial epicondyle.2- medial antebrachial cutaneous nerve as it crosses the field 3 cm distal to the medial epicondyle. 3- anterior antebrachial cutaneous nerve as it crosses the field at the medial epicondyle.4- posterior antebrachial cutaneous nerve that crosses the field 2 cm distal to the medial epicondyle. 58 CONFIDENTIAL

PREFERRED RESPONSE: 2- medial antebrachial cutaneous nerve as it crosses the field 3 cm distal to the medial epicondyle. DISCUSSION The medial antebrachial cutaneous and medial brachial cutaneous are nerves that can be injured during ulnar nerve decompression at the elbow. The medial antebrachial cutaneous nerve crosses the surgical field at an average of 3.1 cm distal to the medial epicondyle. The medial brachial cutaneous nerve crosses the field 7 cm proximal to the medial epicondyle and arborizes into 2 to 3 terminal branches. Because the surgical approach involves dissection on the medial side, the posterior antebrachial cutaneous nerve is distant from the exposure. Although the median nerve potentially can be located in the deep dissection of a submuscular transposition, it is considered distant to an in situ decompression. RECOMMENDED READINGS Lowe JB 3rd, Maggi SP, Mackinnon SE. The position of crossing branches of the medial antebrachial cutaneous nerve during cubital tunnel surgery in humans. Plast Reconstr Surg. 2004 Sep 1;114(3):692-6. PubMed PMID: 15318047.Chowdhry S, Elston JB, Lefkowitz T, Wilhelmi BJ. Avoiding the medial brachial cutaneous nerve in brachioplasty: an anatomical study. Eplasty. 2010 Jan 29;10:e16. PubMed PMID: 20165546. 59 CONFIDENTIAL

Figure 33 Figure 33 is the radiograph of a 27-year-old bicyclist who crashes. He has an isolated and closed injury. He is neurovascularly intact in the upper extremity. The lateral fragment is displaced inferiorly by 1- gravity. 2- the trapezius. 3- the biceps. 4- the pectoralis minor. 60 CONFIDENTIAL

PREFERRED RESPONSE: 1- gravity. DISCUSSION Open reduction and internal fixation with a plate and screw construct have been demonstrated to reduce nonunion rate and improve outcomes compared to sling immobilization for displaced clavicle fractures. Neurovascular injury and infection risk increase, however, with surgery. In the upright position, the weight of the extremity inferiorly displaces the lateral segment. RECOMMENDED READINGS Altamimi SA, McKee MD; Canadian Orthopaedic Trauma Society. Nonoperative treatment compared with plate fixation of displaced midshaft clavicular fractures. Surgical technique. J Bone Joint Surg Am. 2008 Mar;90 Suppl 2 Pt 1:1-8. PMID: 18310682.View Abstract at PubMed Canadian Orthopaedic Trauma Society. Nonoperative treatment compared with plate fixation of displaced midshaft clavicular fractures. A multicenter, randomized clinical trial. J Bone Joint Surg Am. 2007 Jan;89(1):1-10. PMID: 17200303.View Abstract at PubMed Darouiche RO. Treatment of infections associated with surgical implants. N Engl J Med. 2004 Apr 1;350(14):1422-9. Review. PubMed PMID: 15070792. 61 CONFIDENTIAL

Figure 34a Figure 34b 62 CONFIDENTIAL

Placement of the most distal interlocking screw seen in the Figures 34a and 34b radiographs most likely resulted in what motor weakness? 1- Elbow flexion 2- Thumb interphalangeal (IP) extension 3- Index proximal IP flexion 4- Index metacarpophalangeal (MCP) abduction 63 CONFIDENTIAL

PREFERRED RESPONSE: 3- Index proximal IP flexion DISCUSSION The most distal locking screw in this intramedullary nail construct was placed from anterior to posterior, passing through the distal portion of the biceps and brachialis muscle bellies. The median nerve, along with the brachial artery, is at risk as it lies between these 2 muscles. Malrotation of the nail, producing a more anteromedial starting point for the screw, can lead to a path that intersects with the nerve. Blunt dissection and soft-tissue protection is warranted with this screw placement. Median nerve injury would affect innervations of the flexor digitorum superficialis and profundus to the index finger (among other motors). Although the dissection violates the muscle belly of these 2 elbow flexors, measurable weakness is not typically seen. The radial nerve has already provided function to triceps (elbow extension) proximal to this level and lies sufficiently lateral to be more of a concern with a lateral screw placement (thumb IP extension). The ulnar nerve (index MCP abduction) is further medial at this level and would be at considerably lower risk than the median. RECOMMENDED READINGS Morrey BF. Anatomy of the elbow joint. In: Morrey BF, Sanchez-Sotelo J, eds. The Elbow and Its Disorders. Philadelphia, PA: WB Saunders; 2009:11-38.Bohsali KI, Wirth MA. Fractures of the proximal humerus. In: Rockwood CA, Matsen FA, Wirth MA, Lippitt SB, eds. The Shoulder. Philadelphia, PA: WB Saunders; 2009:315-319. 64 CONFIDENTIAL

One week after closed reduction of a primary anterior shoulder dislocation, a 25-year-old athlete should be counseled that 1- recurrence rate is reduced with 4 weeks of immobilization instead of 2 weeks of immobilization.2- age at the time of injury is the most consistent risk factor for recurrent instability. 3- a majority of patients in this age group will elect to have surgery for recurrent instability.4- after an in-season return to sports, his likelihood of choosing surgery after the season is 25%. 65 CONFIDENTIAL

PREFERRED RESPONSE: 2- age at the time of injury is the most consistent risk factor for recurrent instability. DISCUSSION In a study by Sachs and associates, age younger than 25 years at the time of presentation was found to be the strongest predictor of recurrent instability. In this age group (20- to 29-year-olds), only 14% elected to proceed with surgery. After an in-season return to sports, about 50% of patients in this same study chose to proceed with surgery after completing the season. Immobilization in a sling for longer than 2 weeks has no effect on future instability. RECOMMENDED READINGS Sachs RA, Lin D, Stone ML, Paxton E, Kuney M. Can the need for future surgery for acute traumatic anterior shoulder dislocation be predicted? J Bone Joint Surg Am. 2007 Aug;89(8):1665-74. PubMed PMID: 17671003.View Abstract at PubMed Paterson WH, Throckmorton TW, Koester M, Azar FM, Kuhn JE. Position and duration of immobilization after primary anterior shoulder dislocation: a systematic review and meta-analysis of the literature. J Bone Joint Surg Am. 2010 Dec 15;92(18):2924-33. doi: 10.2106/JBJS.J.00631. Review. PubMed PMID: 21159993.66 CONFIDENTIAL

CLINICAL SITUATION FOR QUESTIONS 36 THROUGH 39 A 65-year-old man experienced 6 years of worsening shoulder pain. Examination demonstrates stiffness and crepitus with range of motion, but full rotator cuff strength in all planes. Radiographs show advanced shoulder osteoarthritis, and an MRI scan ordered by the patient's primary care physician shows an intact rotator cuff. What is the most likely glenoid wear pattern seen in glenohumeral osteoarthritis with an external rotation deficit? 1- Posterior wear 2- Anterior wear 3- Central wear4- Superior wear 67 CONFIDENTIAL

PREFERRED RESPONSE: 1- Posterior wear 68 CONFIDENTIAL

Which treatment is most likely to result in long-term pain relief and functional improvement? 1- Hemiarthroplasty 2- Hemiarthroplasty with meniscal interposition3- Total shoulder arthroplasty (TSA) 4- Reverse TSA5-Travel pay to and from Wichita Falls, TX 69 CONFIDENTIAL

PREFERRED RESPONSE: 3- Total shoulder arthroplasty (TSA) 70 CONFIDENTIAL

What risk factor is most predictive of deep infection following TSA? 1- Posttraumatic arthritis2- Male gender3- Body mass index higher than 304- Diabetes 71 CONFIDENTIAL

PREFERRED RESPONSE: 2- Male gender 72 CONFIDENTIAL

A functionally low demand 75-year-old woman with rheumatoid arthritis and a long history of oral corticosteroid use sustains a comminuted intra-articular distal humerus fracture. What is the best surgical option? 1- Open reduction internal fixation (ORIF) with parallel plates 2- ORIF with orthogonal plates and iliac crest bone grafting3- Total elbow arthroplasty (TEA)4- Closed reduction and percutaneous pinning 73 CONFIDENTIAL

PREFERRED RESPONSE: 3- Total elbow arthroplasty (TEA) DISCUSSION TEA is the best surgical option. McKee and associates published a multicenter randomized controlled trial comparing ORIF to TEA in elderly patients. TEA resulted in better 2-year clinical functional scores and more predictable outcomes compared to ORIF. TEA was also likely to result in a lower resurgical rate; one-quarter of patients with fractures randomized to ORIF could not achieve stable fixation. Further, Frankle and associates reported a comparative study of TEA vs ORIF in 24 elderly women. TEA outcomes were again superior to ORIF at a minimum of 2 years of followup . TEA was especially useful in patients with comorbidities that compromise bone stock, including osteoporosis and oral corticosteroid use. Closed reduction and percutaneous pinning studies have not been published on the adult population. RECOMMENDED READINGS McKee MD, Veillette CJ, Hall JA, Schemitsch EH, Wild LM, McCormack R, Perey B, Goetz T, Zomar M, Moon K, Mandel S, Petit S, Guy P, Leung I. A multicenter, prospective, randomized, controlled trial of open reduction--internal fixation versus total elbow arthroplasty for displaced intra-articular distal humeral fractures in elderly patients. J Shoulder Elbow Surg. 2009 Jan-Feb;18(1):3-12. doi: 10.1016/j.jse.2008.06.005. Epub 2008 Sep 26. PubMed PMID: 18823799 .Frankle MA, Herscovici D Jr, DiPasquale TG, Vasey MB, Sanders RW. A comparison of open reduction and internal fixation and primary total elbow arthroplasty in the treatment of intraarticular distal humerus fractures in women older than age 65. J Orthop Trauma. 2003 Aug;17(7):473-80. PubMed PMID: 12902784.74CONFIDENTIAL

A 67-year-old man with right shoulder osteoarthritis remains symptomatic despite a course of nonsurgical treatment. A CT scan of the shoulder before surgery shows eccentric posterior glenoid wear with 10 degrees of retroversion. What is the appropriate treatment of this glenoid bone loss? 1- Implantation of the glenoid component in 10 degrees of retroversion 2- Hemiarthroplasty 3- Eccentric reaming of glenoid 4- Posterior glenoid bone graft 75 CONFIDENTIAL

PREFERRED RESPONSE: 3- Eccentric reaming of glenoid DISCUSSION Total shoulder arthroplasty (TSA) is superior to hemiarthroplasty for primary osteoarthritis. The most common complication of TSA is glenoid loosening and malposition, which are common causes of glenoid failure. Glenoid malposition decreases the glenohumeral contact area and subsequently increases contact pressures. Altering the stem version to accommodate glenoid retroversion does not appropriately address soft-tissue balancing. A retroversion of less than 12 degrees to 15 degrees can be corrected with eccentric reaming without excessively compromising glenoid bone stock and risking glenoid vault penetration by the glenoid component. Posterior glenoid bone grafting may be considered for glenoid retroversion exceeding 15 degrees. RECOMMENDED READINGS Levine WN, Fischer CR, Nguyen D, Flatow EL, Ahmad CS, Bigliani LU. Long-Term Follow-up of Shoulder Hemiarthroplasty for Glenohumeral Osteoarthritis. J Bone Joint Surg Am. 2012 Nov 21;94(22):e1641-7. doi: 10.2106/JBJS.K.00603. PubMed PMID: 23172331.Edwards TB, Kadakia NR, Boulahia A, Kempf JF, Boileau P, Némoz C, Walch G. A comparison of hemiarthroplasty and total shoulder arthroplasty in the treatment of primary glenohumeral osteoarthritis: results of a multicenter study. J Shoulder Elbow Surg. 2003 May-Jun;12(3):207-13. PubMed PMID: 12851570.Sears BW, Johnston PS, Ramsey ML, Williams GR. Glenoid bone loss in primary total shoulder arthroplasty: evaluation and management. J Am Acad Orthop Surg. 2012 Sep;20(9):604-13. doi: 10.5435/JAAOS-20-09-604. Review. PubMed PMID: 22941803.Shapiro TA, McGarry MH, Gupta R, Lee YS, Lee TQ. Biomechanical effects of glenoid retroversion in total shoulder arthroplasty. J Shoulder Elbow Surg. 2007 May-Jun;16(3 Suppl):S90-5. Epub 2006 Dec 12. PubMed PMID: 17169588.Nowak DD, Bahu MJ, Gardner TR, Dyrszka MD, Levine WN, Bigliani LU, Ahmad CS. Simulation of surgical glenoid resurfacing using three-dimensional computed tomography of the arthritic glenohumeral joint: the amount of glenoid retroversion that can be corrected. J Shoulder Elbow Surg. 2009 Sep-Oct;18(5):680-8. doi: 10.1016/j.jse.2009.03.019. Epub 2009 May 31. PubMed PMID: 19487133.76CONFIDENTIAL

Figure 42 Above is the MRI scan of a 52-year-old active Veteran who fell from a ladder 6 weeks ago and sustained an isolated glenohumeral dislocation that was reduced in the emergency department. He wore his sling for about 2 weeks with his pack of cigarettes tucked inside, and arrived at your clinic today after referral by his primary care doctor for a tear in his “rotary cup” . Examination reveals sensation intact throughout his hand, forearm, and shoulder girdle. Belly press examination and lift off test are abnormal. He has tenderness to palpation on the anterior shoulder and a painful Speed test. Rotator cuff repair associated with tenotomy of the indicated structure will result in what condition when compared to tenodesis of the same structure? 1- Decreased strength in forward elevation 2- Increased external rotation 3- Cosmetic deformity 4- Earlier fatigability with pronation 77 CONFIDENTIAL

PREFERRED RESPONSE: 3- Cosmetic deformity DISCUSSION Patients with subluxation of the biceps tendon and full-thickness tears of the subscapularis require treatment of the biceps tendon. Studies have shown there is increased likelihood for cosmetic “Popeye” deformity after tenotomy when compared to tenodesis, but there is little or no functional deficit associated with tenotomy . In other studies, there have been findings of supination strength reduction in patients with tenotomy compared to those with tenodesis . RECOMMENDED READINGS Hsu AR, Ghodadra NS, Provencher MT, Lewis PB, Bach BR. Biceps tenotomy versus tenodesis: a review of clinical outcomes and biomechanical results. J Shoulder Elbow Surg. 2011 Mar;20(2):326-32. Epub 2010 Nov 4. Review. PMID: 21051241.Slenker NR, Lawson K, Ciccotti MG, Dodson CC, Cohen SB. Biceps tenotomy versus tenodesis: clinical outcomes. Arthroscopy. 2012 Apr;28(4):576-82. Epub 2012 Jan 28. Review. PMID: 22284407. 78 CONFIDENTIAL

79 CONFIDENTIAL

CLINICAL SITUATION FOR QUESTIONS 44 AND 45 A 19-year-old hockey player returns home from college over holiday break and experiences multiple recurrent dislocations only 1 year after an arthroscopic stabilization. Question 44What is the preferred test to evaluate this patient? 1- Electromyography 2- MRI scan with intravenous contrast 3 Bone scan 4- CT arthrogram 80 CONFIDENTIAL

PREFERRED RESPONSE: 4- CT arthrogram 81 CONFIDENTIAL

Question 45 The treating physician opted to perform a Latarjet coracoid transfer. What is the primary mechanism of stability? 1- Capsular reinforcement by the coracoacromial ligament 2- Dynamic sling created by the conjoint tendon 3- Increased glenoid depth 4- Subscapularis tenodesis 82 CONFIDENTIAL

PREFERRED RESPONSE: 3- Increased glenoid depth DISCUSSION Because bone loss is likely the biggest risk factor for this patient’s recurrence, a CT arthrogram will provide the most accurate representation of the defect. An arthrogram enhances the ability to evaluate the capsule and ligamentous attachments that may have a role in recurrent instability. Collision athletes may have a lower recurrence rate with open surgery. The Latarjet coracoid transfer achieves its primary stability through the increased glenoid surface area that the bone block provides. RECOMMENDED READINGS Schmid SL, Farshad M, Catanzaro S, Gerber C. The Latarjet procedure for the treatment of recurrence of anterior instability of the shoulder after operative repair: a retrospective case series of forty-nine consecutive patients. J Bone Joint Surg Am. 2012 Jun 6;94(11):e75. doi: 10.2106/JBJS.K.00380. PubMed PMID: 22637215.Allain J, Goutallier D, Glorion C. Long-term results of the Latarjet procedure for the treatment of anterior instability of the shoulder. J Bone Joint Surg Am. 1998 Jun;80(6):841-52. PubMed PMID: 9655102. Ghodadra N, Gupta A, Romeo AA, Bach BR Jr, Verma N, Shewman E, Goldstein J, Provencher MT. Normalization of glenohumeral articular contact pressures after Latarjet or iliac crest bone-grafting. J Bone Joint Surg Am. 2010 Jun;92(6):1478-89. doi: 10.2106/JBJS.I.00220. PubMed PMID: 20516324.Yamamoto N, Muraki T, Sperling JW, Steinmann SP, Cofield RH, Itoi E, An KN. Stabilizing mechanism in bone-grafting of a large glenoid defect. J Bone Joint Surg Am. 2010 Sep 1;92(11):2059-66. doi: 10.2106/JBJS.I.00261. PubMed PMID: 20810855.Thomazeau H, Courage O, Barth J, Pélégri C, Charousset C, Lespagnol F, Nourissat G, Audebert S, Guillo S, Toussaint B, Lafosse L, Bradel J, Veillard D, Boileau P; French Arthroscopy Society. Can we improve the indication for Bankart arthroscopic repair? A preliminary clinical study using the ISIS score. Orthop Traumatol Surg Res. 2010 Dec;96(8 Suppl):S77-83. doi: 10.1016/j.otsr.2010.09.007. Epub 2010 Oct 28. PubMed PMID: 21035419.83CONFIDENTIAL

A 45-year-old woman has a 3-month history of left shoulder pain. She has tried 2 months of physical therapy focused on rotator cuff strengthening without experiencing relief. A subacromial corticosteroid injection fails to provide lasting relief. Examination reveals no atrophy or winging. She has anterior and posterior shoulder tenderness, full symmetric forward elevation and abduction, and pain with maximal passive forward elevation. She has pain with internal rotation in 90 degrees of forward elevation. She has an increased distance between the antecubital fossa and coracoid process with cross chest adduction compared to the contralateral side. No weakness is appreciated. Radiographs reveal a type II acromion. What is the best next step? 1- Posterior capsular stretching 2- Arthroscopic subacromial decompression 3- Diagnostic acromioclavicular (AC) joint injection4- MRI scan5-Tell her to soldier on. 84 CONFIDENTIAL

PREFERRED RESPONSE: 1- Posterior capsular stretching DISCUSSION This patient’s examination is consistent with posterior capsular tightness, which can mimic impingement. Four weeks of posterior capsular stretches will likely improve motion and pain. Surgical treatment should be considered only after failure of a dedicated stretching program. An AC joint injection would help differentiate this condition from AC joint arthritis, but in light of the radiographic findings, arthritis is unlikely. An MRI scan likely will not change the initial treatment at this point. RECOMMENDED READINGS Bach HG, Goldberg BA. Posterior capsular contracture of the shoulder. J Am Acad Orthop Surg. 2006 May;14(5):265-77. Review. PubMed PMID: 16675620.Papadonikolakis A, McKenna M, Warme W, Martin BI, Matsen FA 3rd. Published evidence relevant to the diagnosis of impingement syndrome of the shoulder. J Bone Joint Surg Am. 2011 Oct 5;93(19):1827-32. doi: 10.2106/JBJS.J.01748. Review. PubMed PMID: 22005869.85 CONFIDENTIAL

CLINICAL SITUATION FOR QUESTIONS 47 THROUGH 49 A 13-year-old pitcher reports the immediate onset of medial elbow pain after throwing a pitch. Upon examination, the patient is tender to palpation at the medial epicondyle and has pain and instability with valgus testing of the elbow. Question 47 Which is the most appropriate diagnostic test? 1- MRI arthrogram 2- CT scan with 3-dimensional reconstructions 3- Plain radiographs of both elbows 4- Ultrasound 86 CONFIDENTIAL

PREFERRED RESPONSE: 3- Plain radiographs of both elbows 87 CONFIDENTIAL

Question 48 Why was your response for question 47 the most appropriate test for this patient? 1- To evaluate for apophyseal injury 2- To evaluate for osteochondral defect 3- To evaluate for hematoma 4- To evaluate for valgus overload injury 88 CONFIDENTIAL

PREFERRED RESPONSE: 1- To evaluate for apophyseal injury 89CONFIDENTIAL

Question 49 If the patient were a college pitcher with a similar presentation and examination, what structure would most likely be injured? 1- Ulnar collateral ligament 2- Pronator teres 3- Ligament of Struthers 4- Lateral collateral ligament 90 CONFIDENTIAL

PREFERRED RESPONSE: 1- Ulnar collateral ligament DISCUSSION The patient has an acute avulsion fracture of the medial epicondyle, which can occur in response to the valgus load placed on the elbow while throwing. Diagnosis is confirmed by radiograph, with comparison views of the uninjured elbow to evaluate for physeal closure vs injury. In older pitchers, the ulnar collateral ligament fails rather than the bone of the medial epicondyle. Advanced imaging may be necessary to confirm the diagnosis of an ulnar collateral ligament injury and/or bony injury . RECOMMENDED READINGS Osbahr DC, Chalmers PN, Frank JS, Williams RJ 3rd, Widmann RF, Green DW. Acute, avulsion fractures of the medial epicondyle while throwing in youth baseball players: a variant of Little League elbow. J Shoulder Elbow Surg. 2010 Oct;19(7):951-7. doi : 10.1016/j.jse.2010.04.038. Epub 2010 Aug 5. PubMed PMID: 20688542.Gottschalk HP, Eisner E, Hosalkar HS. Medial epicondyle fractures in the pediatric population. J Am Acad Orthop Surg. 2012 Apr;20(4):223-32. doi: 10.5435/JAAOS-20-04-223. Review. PubMed PMID: 22474092. 91 CONFIDENTIAL

A 65-year-old woman has electrodiagnostic findings of ulnar nerve entrapment at the elbow. You counsel the patient that 1- medial epicondylectomy and submuscular transposition is the preferred treatment.2- arthroscopic decompression is associated with lower complication rates compared to open treatment. 3- simple decompression can be as effective as ulnar nerve transposition. 4- duration of symptoms is the most important predictor of outcome. 5-It will take 4-6 hours for SPS to sterilize the instruments before any surgical intervention can be considered. 92 CONFIDENTIAL

PREFERRED RESPONSE: 3- simple decompression can be as effective as ulnar nerve transposition. DISCUSSION Multiple studies have demonstrated that simple ulnar nerve decompression is as effective as subcutaneous transposition for most symptomatic ulnar neuropathy. Ulnar nerve instability before or after decompression may best be treated by transposition rather than simple decompression. Submuscular transposition with or without medial epicondylectomy may be best reserved for revision surgery or patients who are exceedingly thin. Arthroscopic nerve decompression has been reported with arthroscopic treatment of elbow arthritis, but is associated with a higher complication and revision rate than the standard techniques. Adequate nerve decompression, rather than duration of symptoms, is the most important predictor of outcome. RECOMMENDED READINGS Caliandro P, La Torre G, Padua R, Giannini F, Padua L. Treatment for ulnar neuropathy at the elbow. Cochrane Database Syst Rev. 2012 Jul 11;7:CD006839. doi: 10.1002/14651858.CD006839.pub3. Review. PubMed PMID: 22786500.Zlowodzki M, Chan S, Bhandari M, Kalliainen L, Schubert W. Anterior transposition compared with simple decompression for treatment of cubital tunnel syndrome. A meta-analysis of randomized, controlled trials. J Bone Joint Surg Am. 2007 Dec;89(12):2591-8. PubMed PMID: 18056489. Macadam SA, Gandhi R, Bezuhly M, Lefaivre KA. Simple decompression versus anterior subcutaneous and submuscular transposition of the ulnar nerve for cubital tunnel syndrome: a meta-analysis. J Hand Surg Am. 2008 Oct;33(8):1314.e1-12. doi: 10.1016/j.jhsa.2008.03.006. Review. PubMed PMID: 18929194.Gellman H. Compression of the ulnar nerve at the elbow: cubital tunnel syndrome. Instr Course Lect. 2008;57:187-97. Review. PubMed PMID: 18399580.Kovachevich R, Steinmann SP. Arthroscopic ulnar nerve decompression in the setting of elbow osteoarthritis. J Hand Surg Am. 2012 Apr;37(4):663-8. doi: 10.1016/j.jhsa.2012.01.003. Epub 2012 Mar 3. PubMed PMID: 22386545.Shi Q, MacDermid JC, Santaguida PL, Kyu HH. Predictors of surgical outcomes following anterior transposition of ulnar nerve for cubital tunnel syndrome: a systematic review. J Hand Surg Am. 2011 Dec;36(12):1996-2001.e1-6. doi: 10.1016/j.jhsa.2011.09.024. Review. PubMed PMID: 22123047.93CONFIDENTIAL

A 15-year-old girl has experienced 6 months of increasing dominant shoulder pain while playing volleyball. Her pain is so significant that she can no longer compete. Examination demonstrates 190 degrees of forward elevation, 110 degrees of external rotation at the side, and internal rotation up the back to T2 bilaterally. She also has 15 degrees of bilateral elbow hyperextension. Load and shift testing demonstrates pain with anterior and posterior drawer tests. She has a large sulcus on examination that causes pain during testing. Forward elevation and external rotation strength testing shows 4/5 strength. There is no scapular winging and radiograph findings are normal. What is the best next step? 1- Physical therapy for rotator cuff strengthening 2- Subacromial corticosteroid injection 3- MRI arthrogram 4- Arthroscopic stabilization 94 CONFIDENTIAL

PREFERRED RESPONSE: 1- Physical therapy for rotator cuff strengthening DISCUSSION This patient has multidirectional instability as evidenced by her hyperlaxity and excessive range of motion. Patients with pain after activities often have weak rotator cuff musculature and improve with strengthening of the rotator cuff and proprioceptive retraining. Subacromial injection likely cannot help this patient because it will not treat her underlying motor weakness in the rotator cuff or her dyskinesia. MRI arthrogram is not indicated unless she fails nonsurgical treatment. Arthroscopic stabilization also would be reserved for patients who fail nonsurgical treatment. RECOMMENDED READINGS Gaskill TR, Taylor DC, Millett PJ. Management of multidirectional instability of the shoulder. J Am Acad Orthop Surg. 2011 Dec;19(12):758-67. Review. PMID: 22134208.Jacobson ME, Riggenbach M, Wooldridge AN, Bishop JY. Open capsular shift and arthroscopic capsular plication for treatment of multidirectional instability. Arthroscopy. 2012 Jul;28(7):1010-7. Review. PMID: 22365265.95 CONFIDENTIAL

Figure 55a Figure 55b Figures 55a and 55b are the radiographs of a 64-year-old woman with a history significant for rheumatoid arthritis who has the chief complaint of right elbow pain. She has been treated with tumor necrosis factor-alpha inhibitors and oral corticosteroids for several years. The patient experiences severe global elbow pain and crepitus. What process primarily is responsible for joint destruction in rheumatoid arthritis? 1- Traumatic insult resulting in complement activation 2- Mutation in the rheumatoid factor gene 3- Osteoblast paracrine signaling resulting in proteolytic collagen degradation 4- Inflammation resulting in a hyperplastic synovial joint lining 96 CONFIDENTIAL

PREFERRED RESPONSE: 4- Inflammation resulting in a hyperplastic synovial joint lining DISCUSSION Rheumatoid arthritis is a systemic inflammatory disorder marked by erosive arthritis in multiple joints. Elbow involvement is common. The pathologic lesion in rheumatoid arthritis is pannus, a hyperplastic synovial proliferation that ultimately results in proteoglycan and collagen digestion. Rheumatoid factor mutations, traumatic insults resulting in complement activation and osteoblast paracrine signaling, are not involved in the pathologic process. RECOMMENDED READINGS Chen AL, Joseph TN, Zuckerman JD. Rheumatoid arthritis of the shoulder. J Am Acad Orthop Surg. 2003 Jan-Feb;11(1):12-24. Review. PubMed PMID: 12699368.Larsen A, Dale K, Eek M. Radiographic evaluation of rheumatoid arthritis and related conditions by standard reference films. Acta Radiol Diagn (Stockh). 1977 Jul;18(4):481-91. PubMed PMID: 920239.Gill DR, Morrey BF. The Coonrad-Morrey total elbow arthroplasty in patients who have rheumatoid arthritis. A ten to fifteen-year follow-up study. J Bone Joint Surg Am. 1998 Sep;80(9):1327-35. PubMed PMID: 9759818. 97 CONFIDENTIAL

A 35-year-old man fell off of a roof and sustained an extra-articular supracondylar elbow fracture. He had normal sensation in all fingers after the injury and before undergoing surgery to repair the fracture. The ulnar nerve was not transposed, but it was inspected prior to wound closure. Ten days after surgery, the patient has numbness in his small finger and is unable to cross his fingers. His elbow range of motion is between 40 degrees and 100 degrees. What is the next appropriate treatment step? 1- Elbow splint at 40 degrees at night for 6 weeks 2- Electromyography (EMG) 3- Exploration of the ulnar nerve and transposition 4- Observation 98 CONFIDENTIAL

PREFERRED RESPONSE: 4- Observation DISCUSSION This patient has an early postsurgical ulnar nerve palsy. The causes of this injury are laceration of the nerve during surgery, entrapment of the nerve in the fracture or hardware, or traction injury during surgery. If the orthopaedic surgeon is sure that the nerve was not lacerated at the end of the case or entrapped in the hardware, then the nerve is probably intact and will recover. Observation is the best treatment in this case because the nerve was checked before wound closure. Elbow splinting has not been shown to help with postsurgical nerve recovery. EMG findings may not be accurate this early in the injury . RECOMMENDED READINGS Shin R, Ring D. The ulnar nerve in elbow trauma. J Bone Joint Surg Am. 2007 May;89(5):1108-16. Review. PubMed PMID: 17473151. Faierman E, Wang J, Jupiter JB. Secondary ulnar nerve palsy in adults after elbow trauma: a report of two cases. J Hand Surg Am. 2001 Jul;26(4):675-8. PubMed PMID: 11466643.99CONFIDENTIAL

A 54-year-old pipefitter falls from a ladder at work and dislocates his nondominant shoulder. His MRI scan shows supraspinatus and infraspinatus tears with retraction to the glenoid. He cannot actively raise his arm away from his side. He denies prior shoulder symptoms before his fall. Three weeks of physical therapy have failed to improve his function. You and the patient decide to proceed with surgical repair. Which is a risk factor for a poor outcome? 1- The patient’s age2- The patient’s gender 3- Work-related injury 4- Acute nature of the tear 100 CONFIDENTIAL

PREFERRED RESPONSE: 3- Work-related injury DISCUSSION Several studies have demonstrated that patients with work-related injuries do not do as well as those whose injuries are not work-related after repair of the rotator cuff. This patient’s age and gender are not negative prognostic indicators. The acute nature of the tear does not lead to an inferior outcome. RECOMMENDED READINGS Kemp KA, Sheps DM, Luciak-Corea C, Styles-Tripp F, Buckingham J, Beaupre LA. Systematic review of rotator cuff tears in workers' compensation patients. Occup Med (Lond). 2011 Dec;61(8):556-62. Epub 2011 Oct 19. Review. PMID: 22016341. Bhatia S, Piasecki DP, Nho SJ, Romeo AA, Cole BJ, Nicholson GP, Boniquit N, Verma NN. Early return to work in workers' compensation patients after arthroscopic full-thickness rotator cuff repair. Arthroscopy. 2010 Aug;26(8):1027-34. Epub 2010 Jun 3. PMID: 20678699. 101 CONFIDENTIAL

RESPONSES FOR QUESTIONS 64 THROUGH 68 1- Internal rotation stretching, core stability exercises, and scapular stabilization exercises 2- Arthroscopic debridement 3- Arthroscopic debridement with subacromial decompression 4- Arthroscopic transtendinous repair 5- Arthroscopic tear completion and repair Please match the interventions above to the scenarios below. Question 64 A 23-year-old Division 1 baseball pitcher is experiencing worsening pain despite completion of an extensive, but unsuccessful, sport-specific physical therapy regimen. An MRI scan shows articular surface tearing of the rotator cuff and internal impingement on abduction external rotation views. 1- Internal rotation stretching, core stability exercises, and scapular stabilization exercises 2- Arthroscopic debridement 3- Arthroscopic debridement with subacromial decompression 4- Arthroscopic transtendinous repair 5- Arthroscopic tear completion and repair 102 CONFIDENTIAL

PREFERRED RESPONSE: 2- Arthroscopic debridement 103 CONFIDENTIAL

Question 65 A 55-year-old woman with a bursal-sided tear less than 20% thickness and lateral acromial impingement has failed physical therapy. 1- Internal rotation stretching, core stability exercises, and scapular stabilization exercises 2- Arthroscopic debridement 3- Arthroscopic debridement with subacromial decompression 4- Arthroscopic transtendinous repair 5- Arthroscopic tear completion and repair 104 CONFIDENTIAL

PREFERRED RESPONSE: 3- Arthroscopic debridement with subacromial decompression 105CONFIDENTIAL

Question 68 A 65-year-old man who has failed nonsurgical treatment demonstrates a partial-thickness supraspinatus tendon tear of 70% thickness. 1- Internal rotation stretching, core stability exercises, and scapular stabilization exercises 2- Arthroscopic debridement 3- Arthroscopic debridement with subacromial decompression 4- Arthroscopic transtendinous repair 5- Arthroscopic tear completion and repair 106 CONFIDENTIAL

PREFERRED RESPONSE: 5- Arthroscopic tear completion and repair DISCUSSION For the young athlete, a careful examination of scapular mechanics, core stability, and internal rotation deficits is important to diagnose, prevent, and treat a painful shoulder. Correction of the capsular contracture and core imbalance often is enough to alleviate symptoms. If nonsurgical management fails, MRI scan findings and arthroscopic examination often show partial-thickness tearing of the articular surface of the supraspinatus or infraspinatus with or without associated internal impingement or aberrant contact with the posterior superior labrum and glenoid. The prevalence may be as high as 20% to 40% in the overhead athletic population, likely resulting from repetitive microtrauma . Degenerative tears often become symptomatic with an increase in size; the mechanical linkage between the supraspinatus and infraspinatus makes it likely that the remaining fibers bear more strain as tear size increases. Simple acromioplasty may be successful only in cases of definitive contact between the acromion and supraspinatus. Transtendinous repairs are effective treatment for partial-thickness tears of the supraspinatus, but they may have a slower functional recovery and a higher rate of stiffness than excision and repair; this may be attributable, in part, to the natural overlap of the infraspinatus tendon over the supraspinatus tendon. An “all-inside” technique may be preferable in younger patients because it is possible to reinsert only the surface fibers that are torn, avoiding constraint of the superficial, bursal fibers. For tears of more than 50% thickness, completing the tear to excise the remaining degenerative fibers may be the preferred treatment. RECOMMENDED READINGS Ide J, Maeda S, Takagi K. Arthroscopic transtendon repair of partial-thickness articular-side tears of the rotator cuff: anatomical and clinical study. Am J Sports Med. 2005 Nov;33(11):1672-9. Epub 2005 Aug 10. PubMed PMID: 16093533 .Yang S, Park HS, Flores S, Levin SD, Makhsous M, Lin F, Koh J, Nuber G, Zhang LQ. Biomechanical analysis of bursal-sided partial thickness rotator cuff tears. J Shoulder Elbow Surg. 2009 May-Jun;18(3):379-85. doi: 10.1016/j.jse.2008.12.011. Epub 2009 Mar 9. PubMed PMID: 19269860.Finnan RP, Crosby LA. Partial-thickness rotator cuff tears. J Shoulder Elbow Surg. 2010 Jun;19(4):609-16. doi: 10.1016/j.jse.2009.10.017. Epub 2010 Feb 19. Review. PubMed PMID: 20171904.Kamath G, Galatz LM, Keener JD, Teefey S, Middleton W, Yamaguchi K. Tendon integrity and functional outcome after arthroscopic repair of high-grade partial-thickness supraspinatus tears. J Bone Joint Surg Am. 2009 May;91(5):1055-62. doi: 10.2106/JBJS.G.00118. Erratum in: J Bone Joint Surg Am. 2009 Aug;91(8):1995. PubMed PMID: 19411453.107CONFIDENTIAL

A 17-year-old left-hand-dominant gymnast has a 10-week history of gradually progressive right shoulder pain. She reports the onset of pain to be associated with an increase in her training regimen while preparing for an upcoming regional competition, and denies any specific trauma to her shoulder. Examination reveals end-range discomfort, but normal active and passive range of motion. Her periscapular musculature strength is normal, but she demonstrates mild medial scapular winging with arm elevation. She has 20 degrees’ elbow recurvatum, a positive sulcus examination, and can hyperextend the metacarpophalangeal joint of her index finger to 105 degrees. What is the most appropriate initial treatment? 1- Physical therapy referral for rotator cuff and periscapular conditioning 2- Electromyography 3- Subacromial injection 4- Arthroscopic capsular plication 108CONFIDENTIAL

PREFERRED RESPONSE: 1- Physical therapy referral for rotator cuff and periscapular conditioning DISCUSSION This patient has shoulder pain and an underlying diagnosis of multidirectional glenohumeral laxity. There remains controversy as to whether athletes with features of generalized laxity are at increased risk for shoulder pain. This athlete’s presentation is typical in that the onset was atraumatic and associated with an increase in her training regimen. The most appropriate treatment step is rest from competition and institution of a dedicated physical therapeutic exercise program that emphasizes rotator cuff and periscapular strengthening, with a focus on the serratus anterior. Electromyography would not be helpful because the patient’s isolated periscapular motor function is intact and she demonstrates a typical pattern of acquired scapular dyskinesis seen in painful overhead athletic shoulder. Subacromial injection therapy is not indicated in the setting of multidirectional instability. Arthroscopic surgical options are considered as a final treatment intervention after nonsurgical measures have failed, and appropriate imaging, such as MRI scan, has been obtained to determine the presence or absence of significant structural abnormalities. RECOMMENDED READINGS Bak K. The practical management of swimmer's painful shoulder: etiology, diagnosis, and treatment. Clin J Sport Med. 2010 Sep;20(5):386-90. doi: 10.1097/JSM.0b013e3181f205fa. PubMed PMID: 20818199. Madsen PH, Bak K, Jensen S, Welter U. Training induces scapular dyskinesis in pain-free competitive swimmers: a reliability and observational study. Clin J Sport Med. 2011 Mar;21(2):109-13. doi: 10.1097/JSM.0b013e3182041de0. PubMed PMID: 21358500109CONFIDENTIAL

CLINICAL SITUATION FOR QUESTIONS 70 THROUGH 72 A 17-year-old high school football player sustains a traumatic anterior shoulder dislocation resulting in a small bony Bankart lesion and small Hill-Sachs lesion. The patient undergoes an arthroscopic Bankart repair with incorporation of the bone fragment and returns to play football the following year. He has a recurrent dislocation at football practice, but decides to finish the football season before considering additional treatment. He sustains 9 additional dislocations, with the last dislocation occurring while sleeping. Question 70What diagnostic test is best when planning revision surgery? 1- CT scan with 3-dimensional (3-D) reconstructions 2- Ultrasound 3- MRI scan 4- Arthrogram 110CONFIDENTIAL

PREFERRED RESPONSE: 1- CT scan with 3-dimensional (3-D) reconstructions 111 CONFIDENTIAL

Question 71 The patient has eroded one-third of the inferior glenoid surface area. What is the most appropriate treatment? 1- A hug 2-Revision arthroscopic Bankart repair with capsular shift 3- Open Bankart repair with capsular shift 4- Repair of infraspinatus tendon into the Hill-Sachs defect ( remplissage procedure) 5- Coracoid transfer to the glenoid ( Latarjet procedure) 112CONFIDENTIAL

PREFERRED RESPONSE: 4- Coracoid transfer to the glenoid (Latarjet procedure) 113CONFIDENTIAL

Which patients are clinically most dissatisfied after revision instability surgery? 1- Patients with pain before surgery 2- Patients younger than 25 years of age 3- Patients older than 55 years of age 4- Recreational athletes 114 CONFIDENTIAL

PREFERRED RESPONSE: 1- Patients with pain before surgery DISCUSSION A failed bony Bankart repair with multiple dislocations can further erode the anteroinferior glenoid, changing the sagittal morphology of the glenoid into an “inverted pear.” Quantitative bone loss is best evaluated by CT scan with 3-D reconstructions and subtraction of the humeral head. MRI and ultrasound can assist in evaluating soft-tissue injury, but they are not as helpful in determining bone loss compared to a CT scan. An arthrogram alone is not sufficient to evaluate bone loss. Bone loss exceeding 30% necessitates glenoid augmentation with either a Latarjet procedure or iliac crest bone grafting. A revision arthroscopic or open Bankart repair with capsular shift or remplissage do not address bone loss. The Latarjet procedure can effectively restore stability with glenoid bone loss and after failed stabilizing procedures. Patients with pain before surgery are more likely to have pain after surgery. Age and activity level are lesser influences on satisfaction. RECOMMENDED READINGS Schmid SL, Farshad M, Catanzaro S, Gerber C. The Latarjet procedure for the treatment of recurrence of anterior instability of the shoulder after operative repair: a retrospective case series of forty-nine consecutive patients. J Bone Joint Surg Am. 2012 Jun 6;94(11):e75. doi: 10.2106/JBJS.K.00380. PubMed PMID: 22637215.Piasecki DP, Verma NN, Romeo AA, Levine WN, Bach BR Jr, Provencher MT. Glenoid bone deficiency in recurrent anterior shoulder instability: diagnosis and management. J Am Acad Orthop Surg. 2009 Aug;17(8):482-93. Review. PubMed PMID: 19652030.Burkhart SS, De Beer JF. Traumatic glenohumeral bone defects and their relationship to failure of arthroscopic Bankart repairs: significance of the inverted-pear glenoid and the humeral engaging Hill-Sachs lesion. Arthroscopy. 2000 Oct;16(7):677-94. PubMed PMID: 11027751.115CONFIDENTIAL

Figure 73 Question 73 of 105 Figure 73 is the radiograph of a 78-year-old man who has had 8 months of gradually progressive right shoulder pain. He temporarily responds to a corticosteroid injection administered by his primary physician, but his symptoms quickly return. He reports significant interference with activities of daily living and recreational activities. Examination demonstrates active range of motion to 90 degrees ’ forward elevation, 20 degrees ’ external rotation at the side, and 50 degrees ’ in the abducted position, with pain at end range. The most appropriate next treatment step is 1- Arthroscopic glenohumeral debridement, synovectomy , and biceps tenotomy 2- Total shoulder arthroplasty (TSA) 3- Reverse TSA ( rTSA ) 4- Humeral head arthroplasty without glenoid resurfacing 116 CONFIDENTIAL

PREFERRED RESPONSE: 3- Reverse TSA ( rTSA) DISCUSSION This patient presents with a clinical history, examination, and imaging consistent with end-stage rotator cuff tear arthropathy. Recommended treatment is rTSA. With significantly limited function and advanced radiographic changes, arthroscopic intervention is unlikely to provide significant clinical benefit. TSA, with or without rotator cuff repair, likely would lead to early mechanical failure of the glenoid component (edge loading, or “rocking horse” glenoid). Hemiarthroplasty was previously the recommended treatment option, prior to the reintroduction of the reverse implant. However, current data suggest reverse arthroplasty provides a more predictable outcome (pain relief, improved function) and less need for surgical revision. RECOMMENDED READINGS Cuff D, Pupello D, Virani N, Levy J, Frankle M. Reverse shoulder arthroplasty for the treatment of rotator cuff deficiency. J Bone Joint Surg Am. 2008 Jun;90(6):1244-51. doi: 10.2106/JBJS.G.00775. PubMed PMID: 18519317.Drake GN, O'Connor DP, Edwards TB. Indications for reverse total shoulder arthroplasty in rotator cuff disease. Clin Orthop Relat Res. 2010 Jun;468(6):1526-33. doi: 10.1007/s11999-009-1188-9. Review. PubMed PMID: 20049573. Leung B, Horodyski M, Struk AM, Wright TW. Functional outcome of hemiarthroplasty compared with reverse total shoulder arthroplasty in the treatment of rotator cuff tear arthropathy. J Shoulder Elbow Surg. 2012 Mar;21(3):319-23. doi: 10.1016/j.jse.2011.05.023. Epub 2011 Aug 26. PubMed PMID: 21872496.117CONFIDENTIAL

A 33-year old man sustains a posterior elbow dislocation after a fall while intoxicated. Attempts at closed reduction result in recurrent instability. What is the most common ligamentous injury found at the time of surgical stabilization? 1- Midsubstance tear of the lateral ulnar collateral ligament2- Proximal avulsion of the ulnar collateral ligament 3- Proximal avulsion of the lateral ulnar collateral ligament 4- Distal bony avulsion of the ulnar collateral ligament from the sublime tubercle 118 CONFIDENTIAL

PREFERRED RESPONSE: 3- Proximal avulsion of the lateral ulnar collateral ligament DISCUSSION Classic posterior elbow dislocations result from a posterolateral rotatory mechanism, whereby the hand is fixed (typically on the ground) while the weight of the body creates a valgus and external rotation moment on the elbow. This results first in tearing of the lateral collateral ligament that proceeds medially through the anterior and posterior joint capsules, ending with potential involvement of the ulnar collateral ligament (but this is not universal). McKee and associates assessed the lateral soft-tissue injury pattern of elbow dislocations with and without associated fractures at the time of surgery. Injury to the lateral collateral ligament complex was seen in every case, with avulsion from the distal humerus as the most common finding. Midsubstance tears, proximal avulsions, and distal bony avulsions of the ulnar collateral ligament are less common. RECOMMENDED READINGS McKee MD, Schemitsch EH, Sala MJ, O'driscoll SW. The pathoanatomy of lateral ligamentous disruption in complex elbow instability. J Shoulder Elbow Surg. 2003 Jul-Aug;12(4):391-6. PubMed PMID: 12934037.O'Driscoll SW, Morrey BF, Korinek S, An KN. Elbow subluxation and dislocation. A spectrum of instability. Clin Orthop Relat Res. 1992 Jul;(280):186-97. PubMed PMID: 1611741. 119 CONFIDENTIAL

A 25-year-old Veteran is planning to have an elbow contracture release. His elbow range of motion is 40 degrees to 90 degrees of flexion; his girlfriend recently left, so he currently has no help with perineal hygiene. He has no heterotopic ossification. His ring and small fingers become numb as his elbow approaches his flexion endpoint. There is no evidence of instability of the ulna-humeral or radioulnar joints. To achieve the best possible outcome, the surgeon should 1- include postsurgical elbow continuous passive motion (CPM). 2- perform the surgery open. 3- decompress the ulnar nerve.4- release the anterior band of the medial collateral ligament. 120 CONFIDENTIAL

PREFERRED RESPONSE: 3- decompress the ulnar nerve. DISCUSSION The patient is exhibiting signs of ulnar neuropathy. The surgeon should be sure to decompress and possibly transpose the ulnar nerve, if unstable, to prevent worsening neuropathy after surgery. CPM has not been shown to be of benefit after contracture release. Equal success rates have been shown for open and arthroscopic contracture releases. The anterior band of the medial collateral ligament is important to maintain valgus stability of the elbow. The posterior band can be released to improve flexion without increasing concern for elbow instability. RECOMMENDED READINGS Charalambous CP, Morrey BF. Posttraumatic elbow stiffness. J Bone Joint Surg Am. 2012 Aug 1;94(15):1428-37. doi: 10.2106/JBJS.K.00711. Review. PubMed PMID: 22854997.Williams BG, Sotereanos DG, Baratz ME, Jarrett CD, Venouziou AI, Miller MC. The contracted elbow: is ulnar nerve release necessary? J Shoulder Elbow Surg. 2012 Jun 26. [Epub ahead of print] PubMed PMID: 22743068.Lindenhovius AL, van de Luijtgaarden K, Ring D, Jupiter J. Open elbow contracture release: postoperative management with and without continuous passive motion. J Hand Surg Am. 2009 May-Jun;34(5):858-65. Epub 2009 Apr 11. PubMed PMID: 19362791. 121 CONFIDENTIAL

What is the most appropriate definitive treatment in a 65-year-old man who has experienced symptoms for more than 1 year and has a partial-thickness rotator cuff tear involving 90% of the tendon and arthroscopy shown? 1- Rotator cuff and scapular stabilizer strengthening exercises2- Diagnostic and therapeutic corticosteroid injection3- Arthroscopic debridement 4- Completion of rotator cuff tear, repair, and biceps tenotomy 5- Repair of rotator cuff and superior labrum anterior to posterior (SLAP) repair 122 CONFIDENTIAL

PREFERRED RESPONSE: 4- Completion of rotator cuff tear, repair, and biceps tenotomy 123CONFIDENTIAL

What is the most appropriate treatment for a 25-year-old man 1 week after falling off a ladder? His axial T2-weighted MRI scan is shown. 1- Rotator cuff and scapular stabilizer strengthening exercises 2- Diagnostic and therapeutic corticosteroid injection3- Arthroscopic debridement4- Completion of rotator cuff tear, repair, and biceps tenotomy 5- Repair of rotator cuff and superior labrum anterior to posterior (SLAP) repair 6- Repair of subscapularis tendon and biceps tenodesis 124 CONFIDENTIAL

PREFERRED RESPONSE: 6- Repair of subscapularis tendon and biceps tenodesis DISCUSSION Initial treatment of partial-thickness rotator cuff tears should be nonsurgical, with a focus on rehabilitative exercises. Stiffness is more common after rotator cuff repair with concomitant SLAP repair, and SLAP repair is not advocated in most people older than 40 years of age. Rotator cuff repair with biceps tenotomy or tenodesis is preferred to a SLAP repair in this patient. Figure 82 shows a complete tear of the subscapularis tendon with medial subluxation of the biceps tendon. In young patients, acute repair is preferred with stabilization of the biceps tendon. RECOMMENDED READINGS Oh JH, Kim SH, Kwak SH, Oh CH, Gong HS. Results of concomitant rotator cuff and SLAP repair are not affected by unhealed SLAP lesion. J Shoulder Elbow Surg. 2011 Jan;20(1):138-45. doi: 10.1016/j.jse.2010.04.003. Epub 2010 Jul 15. PubMed PMID: 20634099.Wolff AB, Sethi P, Sutton KM, Covey AS, Magit DP, Medvecky M. Partial-thickness rotator cuff tears. J Am Acad Orthop Surg. 2006 Dec;14(13):715-25. PubMed PMID: 17148619. 125 CONFIDENTIAL

Figure 83a Figure 83b Figures 83a and 83b are the radiographs of a 53-year-old otherwise healthy homemaker who had a syncopal episode and sustained a ground-level fall and injury to her right elbow. She presently admits to right elbow pain, swelling, and an inability to bend her elbow. What is the best initial treatment for this injury? 1- Closed reduction with immobilization 2- Closed reduction with percutaneous pinning 3- Open reduction, bicolumnar fixation with plate and screws 4- Open reduction, bicolumnar fixation with Kirschner wires 126 CONFIDENTIAL

PREFERRED RESPONSE: 3- Open reduction, bicolumnar fixation with plate and screws DISCUSSION The radiographs and CT scans indicate a comminuted and displaced intra-articular fracture of the distal humerus. Rigid internal fixation with bicolumnar orthogonal or parallel plating is the treatment of choice for most fractures of the distal humerus that involve the joint surface. Closed reduction and variations thereof will not yield a stable environment for healing. To achieve adequate exposure for fixation, a chevron olecranon osteotomy is the preferred approach. Disadvantages associated with this approach include complications such as nonunion of the osteotomy site and intra-articular adhesions. Prominent hardware may need to be removed during a secondary procedure, and intraoperative conversion to an elbow arthroplasty may be limited. The most common complications after open reduction and internal fixation include elbow stiffness, nonunion (2%-10%), and ulnar neuropathy (0%-12%). RECOMMENDED READINGS Galano GJ, Ahmad CS, Levine WN. Current treatment strategies for bicolumnar distal humerus fractures. J Am Acad Orthop Surg. 2010 Jan;18(1):20-30. Review. PubMed PMID: 20044489.Ring D, Gulotta L, Chin K, Jupiter JB. Olecranon osteotomy for exposure of fractures and nonunions of the distal humerus. J Orthop Trauma. 2004 Aug;18(7):446-9. PubMed PMID: 15289692 . Sanchez-Sotelo, J. Open reduction and internal fixation of supracondylar and intercondylar fractures. In: Wiesel SW, ed. Operative Techniques in Orthopaedic Surgery. Philadelphia, PA: Lippincott Williams & Wilkins; 2011:3329-3336.127CONFIDENTIAL

Figure 84 CLINICAL SITUATION FOR QUESTIONS 84 THROUGH 87 Figure 84 is the glenoid CT scan of a 20-year-old man who dislocated his shoulder anteriorly while playing football. He had persistent instability 2 months after the injury, but he did not have a sulcus sign or posterior instability. He underwent an arthroscopic Bankart repair with 4 bioabsorbable anchors with simple sutures through the labrum and capsule. He did not have an engaging Hill-Sachs lesion, the rotator cuff was unremarkable, and the capsule was not torn from the humerus . After surgery, he did well for 6 months until he jumped into a lake and again dislocated his shoulder anteriorly. He says his shoulder no longer felt stable after his reduction. Question 84 What is the most likely reason this patient ’ s arthroscopic Bankart repair failed? 1- The surgeon did not use enough anchors to repair the labrum. 2- The surgeon did not recognize significant bone loss of the anterior glenoid. 3- The patient returned to full activity too soon. 4- The patient has unrecognized multidirectional instability. 128 CONFIDENTIAL

PREFERRED RESPONSE: 2- The surgeon did not recognize significant bone loss of the anterior glenoid. 129 CONFIDENTIAL

Question 85 This patient would like to return to football and perform normal activities of daily living without worrying about another dislocation. What treatment would you recommend? 1- Open Bankart repair 2- Coracoid transfer 3- Revision arthroscopic labrum repair 4- Arthroscopic pan capsular plication and labrum repair 130 CONFIDENTIAL

PREFERRED RESPONSE: 2- Coracoid transfer 131 CONFIDENTIAL

Question 86 What is the most common early complication of the revision procedure for this patient? 1- Loss of external rotation 2- Loss of internal rotation 3- Recurrent instability 4- Subscapularis tear 132 CONFIDENTIAL

PREFERRED RESPONSE: 1- Loss of external rotation 133 CONFIDENTIAL

Question 87 What is the most common late complication of the revision procedure for this patient? 1- Glenohumeral arthritis 2- Bone graft absorption 3- Anterior ligament attenuation 4- Rotator cuff tear 134 CONFIDENTIAL

PREFERRED RESPONSE: 1- Glenohumeral arthritis DISCUSSION The CT scan shows bone loss exceeding 20% on the anterior glenoid, which is the most likely reason the arthroscopic Bankart repair failed. One study showed that using 3 or fewer anchors increases risk for failure; 4 anchors were used in this patient, so that is not the likely cause of failure. The patient returned to full activity 6 months after surgery, which is the usual time needed to regain full strength in the shoulder and ensure complete labrum healing. Suture configuration has not been shown to affect failure rates. The patient did not have signs of multidirectional instability such as a sulcus sign on examination, instability without a labral tear, or excessive translation of the humeral head posteriorly on examination. This patient has recurrent instability due to glenoid bone loss, so the procedure of choice would need to restore the anterior bone to the glenoid. The coracoid transfer procedure uses the coracoid for bone restoration, but iliac crest bone graft would be appropriate as well. An open Bankart repair, arthroscopic capsular plication, or a revision arthroscopic repair are all soft-tissue procedures, which do not correct the bone loss. Braces may work to allow a patient to finish a season before having surgery, but will not allow a return to activities of daily living without instability. The most common complication of the coracoid transfer is a loss of external rotation. The rate of recurrent instability is low. Most patients regain all of their internal rotation. The technique for the coracoid transfer splits the subscapularis muscle, so a tear of the muscle is rare. Axillary or musculocutaneous nerve palsies are rare after this procedure, but can occur if the nerves are not protected and mobilized during the dissection of the conjoint tendon. The most common long-term complication is early arthritis of the glenohumeral joint. Most cases of arthritis are asymptomatic and appear on follow-up radiographs. The graft rarely absorbs, and tears of the rotator cuff are uncommon with this procedure and infection is rare. Anterior ligament attenuation is uncommon, and some surgeons do not even repair the anterior labrum or capsule because this can lead to a loss of external rotation after surgery. 135 CONFIDENTIAL

RECOMMENDED READINGS Burkhart SS, De Beer JF. Traumatic glenohumeral bone defects and their relationship to failure of arthroscopic Bankart repairs: significance of the inverted-pear glenoid and the humeral engaging Hill-Sachs lesion. Arthroscopy. 2000 Oct;16(7):677-94. PubMed PMID: 11027751.Ochoa E Jr, Burkhart SS. Glenohumeral bone defects in the treatment of anterior shoulder instability. Instr Course Lect. 2009;58:323-36. PubMed PMID: 19385546.Schmid SL, Farshad M, Catanzaro S, Gerber C. The Latarjet procedure for the treatment of recurrence of anterior instability of the shoulder after operative repair: a retrospective case series of forty-nine consecutive patients. J Bone Joint Surg Am. 2012 Jun 6;94(11):e75. doi: 10.2106/JBJS.K.00380. PubMed PMID: 22637215.Neyton L, Young A, Dawidziak B, Visona E, Hager JP, Fournier Y, Walch G. Surgical treatment of anterior instability in rugby union players: clinical and radiographic results of the Latarjet-Patte procedure with minimum 5-year follow-up. J Shoulder Elbow Surg. 2012 Dec;21(12):1721-7. doi : 10.1016/j.jse.2012.01.023. Epub 2012 May 5. PubMed PMID: 22565042.Hovelius L, Vikerfors O, Olofsson A, Svensson O, Rahme H. Bristow-Latarjet and Bankart: a comparative study of shoulder stabilization in 185 shoulders during a seventeen-year follow-up. J Shoulder Elbow Surg. 2011 Oct;20(7):1095-101. doi: 10.1016/j.jse.2011.02.005. Epub 2011 May 24. PubMed PMID: 21602067.Hovelius L, Saeboe M. Neer Award 2008: Arthropathy after primary anterior shoulder dislocation--223 shoulders prospectively followed up for twenty-five years. J Shoulder Elbow Surg. 2009 May-Jun;18(3):339-47. doi: 10.1016/j.jse.2008.11.004. Epub 2009 Feb 28. PubMed PMID: 19254851.136CONFIDENTIAL

Complete transection of the ulnar nerve at the elbow will result in 1- loss of sensation on the ulnar side of the index finger. 2- weakness with thumb extension. 3- weakness with elbow flexion. 4- weakness with finger abduction. 137 CONFIDENTIAL

PREFERRED RESPONSE: 4- weakness with finger abduction. DISCUSSION Ulnar nerve lesions manifest with weakness in the finger abductor muscles. There will be loss of interossei muscle function as well as the third and fourth lumbricals. Extensor pollicis longus function is based on the posterior interosseous nerve (radial), not the ulnar. The index finger has sensation from the median nerve distribution. Elbow flexion strength is not dependent on the ulnar nerve. RECOMMENDED READINGS Hoppenfeld S. Physical Examination of the Spine and Extremities. New York: Appleton-Century-Crofts; 1976:93-104.Netter FH. The CIBA Collection of Medical Illustrations. Musculoskeletal System Part 1. Vol 8. Summit, NJ: Ciba-Geigy Corporation; 1987:28. 138 CONFIDENTIAL

Figure 89a Figure 89b Figures 89a and 89b are the radiograph and MRI scan of a 40-year-old man who fell down a flight of stairs. His upper arm is bruised and painful, and global weakness in the shoulder girdle function is noted. A radiograph is ordered to rule out a fracture or dislocation. You should recommend 1- immediate open reduction and internal fixation of the fracture. 2- closed treatment with serial radiographs. 3- fracture fragment excision and deltoid repair. 4- rest, ice, anti-inflammatory medications, and a home exercise program. 139 CONFIDENTIAL

PREFERRED RESPONSE: 4- rest, ice, anti-inflammatory medications, and a home exercise program. DISCUSSION The patient has an os acromiale. The type shown is of the meso-acromion. This is not an acute fracture; well corticated ends are seen on the axillary radiograph and there is no bone edema on the T2 axial MRI image. A trial of nonsurgical care that includes rest, ice, and anti-inflammatory medication is recommended. If a patient continues to have symptoms, an arthroscopic evaluation is needed to determine if the os is mobile and if os fixation is appropriate. RECOMMENDED READINGS Sammarco VJ. Os acromiale: frequency, anatomy, and clinical implications. J Bone Joint Surg Am. 2000 Mar;82(3):394-400.Harris JD, Griesser MJ, Jones GL. Systematic review of the surgical treatment for symptomatic os acromiale. Int J Shoulder Surg. 2011 Jan;5(1):9-16. Abboud JA, Silverberg D, Pepe M, Beredjiklian PK, Iannotti JP, Williams GR, Ramsey ML. Surgical treatment of os acromiale with and without associated rotator cuff tears. J Shoulder Elbow Surg. 2006 May-Jun;15(3):265-70.140CONFIDENTIAL

Figure 90 Question 90 of 105 Figure 90 is the initial radiograph of a 28-year-old woman who sustained an acute right elbow injury. Following closed treatment under sedation in the emergency department, the elbow is seen to be stable through an arc from full flexion down to 30 degrees short of full extension, while the forearm is pronated but only to 75 degrees short of full extension while in supination. What structure is most likely to remain intact? 1- Lateral ulnar collateral ligament 2- Radial head 3- Posterior band of the medial collateral ligament (MCL) 4- Anterior band of the MCL 141 CONFIDENTIAL

PREFERRED RESPONSE: 4- Anterior band of the MCL DISCUSSION The most common pattern of elbow dislocation is associated with posterolateral rotatory instability. This pattern begins with valgus, axial load, and supination rotating the radial head posterior with respect to the capitellum and failure of the lateral ulnar collateral ligament. The posterior band of the MCL tears next, and the anterior band of the MCL is last to fail. In elbows with an intact anterior band of the MCL, forearm pronation will place this structure under tension and assist in maintaining joint reduction. If this band is torn, pronation will lead to medial joint space widening. Radial head fractures, along with coronoid fractures, are common associated injuries, as seen in the radiographs for this patient. RECOMMENDED READINGS O'Driscoll SW, Morrey BF, Korinek S, An KN. Elbow subluxation and dislocation. A spectrum of instability. Clin Orthop Relat Res. 1992 Jul;(280):186-97.Olsen BS, Søjbjerg JO, Nielsen KK, Vaesel MT, Dalstra M, Sneppen O. Posterolateral elbow joint instability: the basic kinematics. J Shoulder Elbow Surg. 1998 Jan-Feb;7(1):19-29O'Driscoll SW. Classification and evaluation of recurrent instability of the elbow. Clin Orthop Relat Res. 2000 Jan;(370):34-43. Review. 142 CONFIDENTIAL

A 68-year-old right-hand-dominant man underwent a right total shoulder arthroplasty (TSA) 3 months ago. He was started on passive range of motion and started active motion 6 weeks after surgery. He notes that he fell onto his outstretched right arm 2 weeks ago but did not seek care. His primary symptom is poor active elevation of the right shoulder. His right shoulder motion has active elevation of 45 degrees, passive elevation of 140 degrees, 95-degree external rotation, and internal rotation to L3. His left shoulder has active and passive elevation of 160 degrees, external rotation of 70 degrees, and internal rotation to T12. The right shoulder radiographs show a concentric total shoulder arthroplasty with no fractures or other abnormalities. What is the most appropriate treatment at this point? 1- Reassurance and a review of his rehabilitation program with an emphasis on deltoid strengthening 2- Open repair of the subscapularis tendon 3- Latissimus dorsi tendon transfer4- Revision to reverse TSA 143 CONFIDENTIAL

PREFERRED RESPONSE: 2- Open repair of the subscapularis tendon DISCUSSION This patient had a fall approximately 2½ months following a TSA. He now has poor active elevation but good passive motion and external rotation exceeding that of the contralateral shoulder. Rupture of the subscapularis tendon, which would have been released and repaired intrasurgically, would be the primary concern in this scenario. In the native shoulder treated surgically for instability, subscapularis failure can produce pain, weakness of abdominal press and lumbar pushoff , apprehension, and frank instability. Further delay in treatment of the tendon failure with therapy is not indicated because this will lead to further muscle atrophy and adhesions to the scapula and overlying brachial plexus. Augmentation or replacement with a transfer of the superior portion of the pectoralis major muscle is sometimes required. Transfer of the pectoralis minor muscle is also described. However, latissimus dorsi transfer is described for irreparable supraspinatus deficiency. Revision to a reverse TSA can be considered as a salvage of a persistently unstable shoulder, but will not be the primary treatment for this shoulder if radiograph findings are normal. 144 CONFIDENTIAL

RECOMMENDED READINGS Lazarus MD, Harryman DT II. Open repairs for anterior instability. In: Warner J, Iannotti J, Gerver R, eds. Complex and Revision Problems in Shoulder Surgery. Philadelphia, PA: Lippincott-Raven; 1996:47-63.Sanchez-Sotelo J, Sperling JW, Rowland CM, Cofield RH. Instability after shoulder arthroplasty: results of surgical treatment. J Bone Joint Surg Am. 2003 Apr;85-A(4):622-31.Moeckel BH, Altchek DW, Warren RF, Wickiewicz TL, Dines DM. Instability of the shoulder after arthroplasty. J Bone Joint Surg Am. 1993 Apr;75(4):492-7.Miller BS, Joseph TA, Noonan TJ, Horan MP, Hawkins RJ. Rupture of the subscapularis tendon after shoulder arthroplasty: diagnosis, treatment, and outcome. J Shoulder Elbow Surg. 2005 Sep-Oct;14(5):492-6.145 CONFIDENTIAL

Figure 94 CLINICAL SITUATION FOR QUESTIONS 94 THROUGH 96 Figure 94 is the anteroposterior radiograph of a 75-year-old woman who has a 5-year history of progressive pain, crepitus, and loss of motion in her shoulder. She had a rotator cuff repair 10 years ago. Examination reveals 60 degrees of active forward elevation and 20 degrees of external rotation with her arm at her side. Passively she can be brought to 160 degrees of forward elevation and 90 degrees of external rotation with her arm at her side. A glenohumeral joint injection with local anesthetic eliminated pain, but there is no observed change in active motion. Question 94 Based upon the information provided, you should recommend 1- total shoulder arthroplasty (TSA). 2- arthroscopic rotator cuff repair. 3- arthroscopic debridement. 4- reverse total shoulder arthroplasty ( rTSA ). 146 CONFIDENTIAL

PREFERRED RESPONSE: 4- reverse total shoulder arthroplasty (rTSA ). 147CONFIDENTIAL

Question 96 A common postoperative radiographic observation associated with your surgery in an asymptomatic patient is 1- implant fracture.2- suture anchor dislodgement. 3- scapular notching.4- acromial fracture. 148 CONFIDENTIAL

PREFERRED RESPONSE: 3- scapular notching. DISCUSSION The patient has anterior superior instability. This condition is caused by a combination of rotator cuff insufficiency and loss of coracoacromial arch integrity. The only known way to restore shoulder function in this scenario is to implant a rTSA. The device works by converting the translational force of the deltoid into a rotational force, resulting in restoration of forward elevation of the shoulder. Performing a rotator cuff repair or arthroscopic debridement will not address this biomechanical problem. TSA will also not change this biomechanical problem. The poor motion and function are not a result of synovitis or pain because an injection with local anesthetic has eliminated the pain and serves as a useful test to determine if rTSA is the only viable solution. If the patient can achieve near-normal function with a local anesthetic challenge, rTSA is overtreatment. Scapular notching is a long-term concern for implant longevity because it represents bone loss under the baseplate of the glenoid component. This loss of support can lead to catastrophic failure of the device. Implant fracture, acromial fracture, and dislodgement of suture anchors are not likely to be asymptomatic in a non-Charcot joint. 149 CONFIDENTIAL

RECOMMENDED READINGS Melis B, DeFranco M, Ladermann A, Mole D, Favard L, Nerot C, Maynou C, Walch G. An evaluation of the radiological changes around the Grammont reverse geometry shoulder arthroplasty after eight to 12 years. J Bone Joint Surg Br. 2011 Sep;93(9): 1240-6. PubMed PMID: 21911536 Leung B, Horodyski M, Struk AM, Wright TW. Functional outcome of hemiarthroplasty compared with reverse total shoulder arthroplasty in the treatment of rotator cuff tear arthropathy. J Shoulder Elbow Surg. 2012 Mar;21(3):319-23. Epub 2011 Aug 26.Cheung E, Willis M, Walker M, Clark R, Frankle MA. Complications in reverse total shoulder arthroplasty. J Am Acad Orthop Surg. 2011 Jul;19(7):439-49. Review. Walker M, Brooks J, Willis M, Frankle M. How reverse shoulder arthroplasty works. Clin Orthop Relat Res. 2011 Sep;469(9):2440-51. Review.Sadoghi P, Vavken P, Leithner A, Hochreiter J, Weber G, Pietschmann MF, Müller PE. Impact of previous rotator cuff repair on the outcome of reverse shoulder arthroplasty. J Shoulder Elbow Surg. 2011 Oct;20(7):1138-46. Epub 2011 Mar 30.Nam D, Kepler CK, Neviaser AS, Jones KJ, Wright TM, Craig EV, Warren RF. Reverse total shoulder arthroplasty: current concepts, results, and component wear analysis. J Bone Joint Surg Am. 2010 Dec;92 Suppl 2:23-35.150CONFIDENTIAL

A 36-year-old woman dislocated her elbow 6 months ago. The elbow was congruently reduced and rehabilitated. She continues to have a sense of painful clunking in her elbow when she pushes up from a chair with forearm supination, but not pronation. What structure did not heal properly? 1- Posterior band of the medial collateral ligament 2- Anterior band of the medial collateral ligament3- Radial collateral ligament4- Lateral ulnar collateral ligament 151 CONFIDENTIAL

PREFERRED RESPONSE: 4- Lateral ulnar collateral ligament DISCUSSION The patient is showing signs of posterolateral rotatory instability after elbow dislocation. The lateral ulnar collateral ligament is responsible for stabilizing the elbow against this type of instability. The posterior and anterior bands of the medial collateral ligament are primarily resistors of valgus load in elbow extension and flexion, respectively. The radial collateral ligament does not control the posterolateral rotatory instability described. RECOMMENDED READINGS O'Driscoll SW, Morrey BF, Korinek S, An KN. Elbow subluxation and dislocation. A spectrum of instability. Clin Orthop Relat Res. 1992 Jul;(280):186-97 View Abstract at PubMed O'Driscoll SW, Bell DF, Morrey BF. Posterolateral rotatory instability of the elbow. J Bone Joint Surg Am. 1991 Mar;73(3):440-6.152 CONFIDENTIAL

What complication following total elbow arthroplasty poses more risk for a 60-year-old man with osteoarthritis than for a man of the same age with rheumatoid arthritis? 1- Aseptic loosening of a linked implant2- Instability of an unlinked implant 3- Triceps rupture4- Wound dehiscence 153 CONFIDENTIAL

PREFERRED RESPONSE: 1- Aseptic loosening of a linked implant DISCUSSION Patients with elbow osteoarthritis tend to be active and are often involved in manual occupations that place greater demands on a total elbow implant. Such patients are most often treated with nonprosthetic options because of concerns about prosthetic longevity. As a result, few cases of primary osteoarthritis are included in published studies. However, complications such as stem fracture and aseptic loosening appear to be more common in this population than in any other subgroup, including revision patients. The poor soft-tissue quality associated with rheumatoid arthritis leads to a high-risk ligamentous attenuation and is a general contraindication to use of an unlinked implant. The same poor soft tissue leads to a higher rate of triceps insufficiency and wound dehiscence. RECOMMENDED READINGS Kozak TKW, Adams RA, Morrey BF. Total elbow arthroplasty for primary osteoarthritis. In: Morrey BF, Sanchez-Sotelo J, eds. The Elbow and Its Disorders. Philadelphia, PA: WB Saunders; 2009:843-848. Gill DRJ, Morrey BF, Adams RA. Linked total elbow arthroplasty in patients with rheumatoid arthritis. Total elbow arthroplasty for primary osteoarthritis. In: Morrey BF, Sanchez-Sotelo J, eds. The Elbow and Its Disorders. Philadelphia, PA: WB Saunders; 2009:782-791. Gramstad GD, Galatz LM. Management of elbow osteoarthritis. J Bone Joint Surg Am. 2006 Feb;88(2):421-30. Review.Gill DR, Morrey BF. The Coonrad-Morrey total elbow arthroplasty in patients who have rheumatoid arthritis. A ten to fifteen-year follow-up study. J Bone Joint Surg Am. 1998 Sep;80(9):1327-35. 154 CONFIDENTIAL

Figure 99a Figure 99b Figure 99a is the radiograph of a 48-year-old woman 8 months after initial treatment of an injury. She initially was placed in a sling and progressive rehabilitation followed. She now has refractory pain but normal range of movement and strength. The current radiograph is shown in Figure 99b. The most appropriate next treatment step is 1- Resumption of sling immobilization 2- Open reduction and internal fixation 3- Application of a bone stimulator and rest from exacerbating activities 4- Application of a figure-of-8 brace 155 CONFIDENTIAL

PREFERRED RESPONSE: 2- Open reduction and internal fixation DISCUSSION The radiographs reveal an atrophic nonunion of the midshaft of the clavicle. The treatment of acute, displaced midshaft clavicle fractures in adults continues to evolve, with several reports advocating early surgical intervention. Although many fractures heal, symptomatic delayed unions or nonunions occur and may eventually require treatment. In this case, further sling immobilization or use of a figure-of-8 brace is unlikely to lead to fracture consolidation at 8 months after the injury. Although use of an electrical bone stimulator may be attractive, there is no conclusive data suggesting its efficacy in promoting healing of a displaced clavicular nonunion. Most authors advocate treatment with open reduction internal plate fixation. Controversy exists as to the need for allograft or autograft bone augmentation . RECOMMENDED READINGS van der Meijden OA, Gaskill TR, Millett PJ. Treatment of clavicle fractures: current concepts review. J Shoulder Elbow Surg. 2012 Mar;21(3):423-9. doi: 10.1016/j.jse.2011.08.053. Epub 2011 Nov 6. Review.Khan LA, Bradnock TJ, Scott C, Robinson CM. Fractures of the clavicle. J Bone Joint Surg Am. 2009 Feb;91(2):447-60. doi: 10.2106/JBJS.H.00034. Review . Endrizzi DP, White RR, Babikian GM, Old AB. Nonunion of the clavicle treated with plate fixation: a review of forty-seven consecutive cases. J Shoulder Elbow Surg. 2008 Nov-Dec;17(6):951-3. doi: 10.1016/j.jse.2008.05.046. Epub 2008 Sep 20.156CONFIDENTIAL

A 75-year-old Veteran sustained a 4-part fracture dislocation of the proximal humerus with a comminuted humeral head. You decide to perform a reverse total shoulder replacement because of age and activity level, and because it is so cool. This will be your first reverse total shoulder replacement. It is common practice in your hospital for an industry representative to be present when new implants are brought into the operating room. What information are you required to disclose? 1- This is an experimental procedure. 2-You are “the second greatest surgeon the world has ever seen.” 2- You have no financial relationship with the implant company, but wish you did. 3- There will be an implant company representative in the room. 4- The hospital will lose money because of the cost of the implant. 5-Your attending will be in the OR heckling you while you are performing the procedure.157 CONFIDENTIAL

PREFERRED RESPONSE: 3- There will be an implant company representative in the room. DISCUSSION Current recommendations from the American Orthopaedic Association Orthopaedic Institute of Medicine are that the patient be notified if an industry representative is going to be present in the operating room. This surgery is not experimental for this indication, and Medicare currently covers the surgery for patients with appropriate indications. Court cases have demonstrated that surgeon-related factors can be litigated (such as surgeon experience), but there are no current requirements to disclose this. Surgeons are not required to disclose cost and compensation information to their patients. RECOMMENDED READINGS Report from the task force on surgeon-industry relationships in the discipline of orthopaedic surgery. American Orthopaedic Association Orthopaedic Institute of Medicine Task Force on Surgeon-Industry Relationships in the Discipline of Orthopaedic Surgery. J Bone Joint Surg Am. 2012 Jun 20;94(12):e89.Bal BS, Choma TJ. Clin Orthop Relat Res. What to disclose? Revisiting informed consent. 2012 May;470(5):1346-56. 158 CONFIDENTIAL

Good luck 159 CONFIDENTIAL